0% found this document useful (0 votes)
43 views114 pages

NMAT 2022 Mock Test 3

The document discusses a study that found transporting food from production to consumption accounts for 6% of global greenhouse gas emissions, triple previous estimates. Richer countries accounted for 46% of emissions but only 12.5% of the population. Fruit and vegetables had the highest emissions due to refrigeration and out-of-season demand.

Uploaded by

Sunil K
Copyright
© © All Rights Reserved
We take content rights seriously. If you suspect this is your content, claim it here.
Available Formats
Download as PDF, TXT or read online on Scribd
0% found this document useful (0 votes)
43 views114 pages

NMAT 2022 Mock Test 3

The document discusses a study that found transporting food from production to consumption accounts for 6% of global greenhouse gas emissions, triple previous estimates. Richer countries accounted for 46% of emissions but only 12.5% of the population. Fruit and vegetables had the highest emissions due to refrigeration and out-of-season demand.

Uploaded by

Sunil K
Copyright
© © All Rights Reserved
We take content rights seriously. If you suspect this is your content, claim it here.
Available Formats
Download as PDF, TXT or read online on Scribd
You are on page 1/ 114

www.byjusexamprep.

com

Mock Test Solutions in English

Questions

1. Direction: Read the passage given below, and answer the question that follows:

The summer of 1941 was a bleak time for Britain and her European allies in the war. The Nazis were
bombing key cities across the country in what came to be known as the Blitz, and much of Europe had
fallen to the Germans, leaving Britain vulnerable. This made the work of the Special Operations Executive
(SOE), and the actions of a few key women within it, that much more important.

Established in June 1940 in London, the SOE was a volunteer force set up to wage a secret war behind
enemy lines. Prime Minister Winston Churchill famously ordered the SOE’s agents to ‘set Europe ablaze’
through espionage, sabotage, and building a resistance network in occupied Europe. Dozens of women
were recruited by the SOE and deployed as spies, including American agent Virginia Hall.

Considered ‘the most dangerous of all Allied spies’ by the Gestapo (German state secret police), Virginia
Hall hailed from a wealthy family in Baltimore. She ‘took pleasure in defying convention,’ writes Sonia
Purnell in her acclaimed biography, A Woman of No Importance: The Untold Story of Virginia Hall, WW2’s
Most Dangerous Spy, recounting how a young Virginia was ambitious and set her sights on studying in
Paris after attending George Washington University.

On a hunting trip in Turkey in 1933, Hall tripped and accidentally shot herself in the foot; the leg had to be
amputated below the knee, and she used a wooden limb, which she nicknamed ‘Cuthbert’. Despite
gaining experience in various clerk roles at consulates across Europe and having fluency in foreign
languages, Hall’s multiple attempts to become a diplomat in her own right with the State Department were
all rejected on account of her being both a woman and disabled.

Frustrated, she resigned from her administrative job at the U.S. embassy in Estonia and volunteered in
France as an ambulance driver in 1940, before the U.S. officially entered the war. A chance encounter
with an undercover agent led to a connection with the newly formed SOE, and in August 1941, after
training in Britain, Hall went to France under the guise of a reporter with the New York Post named Brigitte
LeContre based in Lyon, becoming the first female agent to take up residence in France.
www.byjusexamprep.com

Her status as a journalist and her charming manner meant that she quickly established a network of
contacts and was able to recruit more people to the Resistance. Under the codename ‘Germaine,’ Hall
also planned jailbreaks for other agents who had been captured, and in all, survived alone in the field for
12 months before other women spies were deployed to France. “This ‘gallant lady,’ her SOE commanders
concluded, was almost single-handedly changing minds about the role of women in combat,” writes
Purnell.

Hall’s activities in galvanising the Resistance were successful, and had attracted the attention of the
French Vichy Police and the Gestapo; but due to her adept use of disguises, the authorities never quite
worked out exactly who ‘Germaine’ was. When the Germans moved to fully occupy France in late 1942,
Hall sensed danger and escaped the Nazis by trekking a 7,500-foot pass through the Pyrenees to Spain
despite being in incredible discomfort due to her prosthetic leg.

Hall then returned to London and continued her espionage work with the American Office of Strategic
Services (OSS), the SOE’s American counterpart. Hall’s work with the OSS in 1944 and 1945 directly led
to the liberation of large parts of France through her leadership of Resistance units, and she later became
the only civilian woman in the war to receive the Distinguished Service Cross for “extraordinary heroism.”

What did Virginia Hall do before working as a spy for SOE?

I. Reporter with the New York Post

II. Ambulance driver in France

III. Diplomat in State Department

IV. Clerical roles in Europe


A. I, II, and IV B. II and IV
C. II, III, and IV D. Only IV
E. Only III
2. Which of the following did Virginia Hall not do as an undercover agent in France?
A. She planned the escape of other undercover B. She worked successfully towards galvanising the
agents who had been captured in jails. resistance in France.
C. She enabled deliveries of arms and explosives to D. She recruited more people to the Resistance by
the resistance network. establishing a network of contacts.
www.byjusexamprep.com

E. She trekked a 7,500-foot pass through the


Pyrenees to Spain despite being disabled.
3. Why was Virginia Hall not recruited as a diplomat with the State Department?
A. Because she was doubted to be an undercover
B. Because she was a disabled woman
agent
C. Because her earlier experience as a clerk was not D. Because she was not very fluent in foreign
enough to become a diplomat languages
E. Because she was being suspected of being a spy
4. All of the following are correct about Virginia Hall except:
A. Virginia was awarded the Distinguished Service B. Virginia used the codename ‘Germaine’ when she
Cross for her ‘extraordinary heroism’. worked as a spy in France.
D. Virginia's left leg was amputated below the knee
C. A young Virginia wanted to study in Paris.
because she was shot in the leg.
E. Virginia was ambitious when she was young.
5. Direction: Read the passage given below and answer the questions that follow:

Transporting food from where it is produced to our dinner plates creates at least triple the amount of
greenhouse gas emissions as previously estimated, a new study suggests.

So-called ‘food miles’ are likely responsible for about 6% of the world’s greenhouse gas emissions, the
authors of the study found after calculating that 3bn tonnes of CO2-equivalent was produced in
transporting food for human consumption each year.

Scientists who carried out the research, which analysed 74 countries and regions and 37 different types of
food, found richer countries accounted for 46% of the emissions from food miles but just 12.5% of the
world’s population.

Fruit and vegetables had the highest food miles emissions because they often needed to be refrigerated
and consumers demanded out-of-season foods.

The University of Sydney’s Prof David Raubenheimer, a co-author of the study published in the journal
Nature Food, said research on making food production more sustainable had tended to focus on
comparing plant-based diets to meat.
www.byjusexamprep.com

“We tend to interpret information around us in simplistic terms, like ‘meat is bad and vegetables are good’
but we wanted a much more comprehensive picture,” he said.

“Our study shows that in addition to shifting towards a plant-based diet, eating locally is ideal, especially in
affluent countries.”

Study leader Dr Mengyu Li, also of the University of Sydney, said when food miles were added to
emissions from land use, production and methane from cattle, about 30% of the world’s greenhouse gas
emissions were linked to food production.

“Food transport emissions add up to nearly half of direct emissions from road vehicles,” she said.

While meat production emits 2.8bn tonnes CO2-e, which is seven times as much as fruit and vegetables,
the emissions from food miles for meat was only 110m tonnes globally.

This compared to 1.06bn tonnes for fruit and vegetables, with the second highest emitting food group
being cereals and flour.

The global total of 3bn tonnes of CO2-e related to food miles was between 3.5 and 7.5 times higher than
previous estimates, the authors said.

As well as calculating the emissions from moving the food to the consumer, the study also included the
emissions from inputs used to produce the food. For example, the calculations would include emissions
from grain that is grown and then transported to feed animals.

About 1.7bn tonnes CO2-e of emissions came from moving food within countries, with 1.3bn tonnes
coming from international transportation.

What is meant by the term ‘food miles’ as used in the passage?


A. The number of miles food has to travel before B. The number of miles food has to cover before
reaching a warehouse reaching the consumers
C. The number of miles fruits and vegetables cover
D. The number of miles food travels during its entire
to the total number of miles covered by other food
lifetime
items to reach their destination
E. The total time food takes to reach the doorsteps
www.byjusexamprep.com

of the retailers
6. The numbers surrounding richer nations and their carbon emissions from food miles imply that:
A. Richer nations don’t know how to transport food B. Richer nations are worse at setting up supply
efficiently. chains for food transportation.
C. Richer nations have made a huge contribution to D. Richer nations have been at the forefront of the
food miles emission numbers. worsening climate crisis.
E. Richer nations are trying to escape blame for
producing greenhouse gases.
7. What is the main idea of the author behind writing this passage?
A. Food miles impact the climate more than B. There is a clear demarcation between different
anticipated. food categories and their carbon emissions.
C. Richer nations must restrict emissions from food D. The situation surrounding food transportation and
transportation. carbon emissions is grave.
E. A unique method of food delivery should be
devised.
8. Which of the following could be the one consumer habit that can significantly reduce carbon emissions
associated with food transportation?
A. Utilisation of freshly produced fruits and B. Penchant for off-season fruits and vegetables is
vegetables at the earliest significantly reduced
C. Tendency to consume fruits and vegetables if
D. Less inclination towards meat and dairy products
they are fresh
E. Storage of fruits all by themselves
9. Direction: Read the passage given below, and answer the questions that follow:

Over 500 trains were cancelled yesterday following protests against the Agnipath scheme. The
cancellation follows instances of arson and wilful destruction of railway property last week. Yet again,
Indian Railways (IR) has been made collateral damage. In the pre-pandemic period, IR carried about 22
million passengers a day, a measure of the scale of its importance. Cancellations not only directly
inconvenience millions but undermine the economy as railway rakes transport 3.3 million tons of freight
daily, mainly coal. This senseless destruction and related disruption should not be shrugged off.

IR is the lightning rod of protests primarily because of its importance. Disrupting railway service attracts
wide attention. While IR is unconnected to the cause of violence, can it do better in protecting its assets?
www.byjusexamprep.com

IR’s security is overseen by its own force, Railway Protection Force (RPF), and backed by central
legislation. As trains crisscross states, state governments also come into the picture. Government Railway
Police (GRP), funded equally by IR and states but under the administrative control of the local
government, and local police assist in safeguarding railway assets and passengers.

RPF does not lack attention. It is not only empowered to arrest without a magistrate’s order, in 2019
handpicked personnel were given commando training. If IR’s security performance is still unsatisfactory, it
is due to gaps in coordination with the state police. This was examined in detail in a 2011 CAG
performance audit of IR’s security. GRP and state police forces handle the security of trains, tracks and
railway premises, while RPF protects properties and consignments. The performance of local policing is
influenced by a state government’s political signalling. Consequently, the leeway agitations have a direct
impact on IR’s assets.

In this unfavourable environment, CAG’s report observed that a unified overseeing arrangement between
RPF and state police was missing. In short, there are coordination problems that IR has not addressed.
Given last week’s destruction, it is clear that the coordination gaps remain unplugged. Their cost will be
borne by people who rely on IR. Between FY-2016 and FY-2020, losses on passenger service rose from
Rs. 36,286 crores to Rs. 63,364 crores. Given GoI’s fiscal challenges and limited rail budgets, willy-nilly
people dependent on train travel will suffer. RPF, headed by a police officer on deputation, needs to
strengthen its coordination protocols with local police. Trains cannot be such easy targets for arsonists.

Which of the following is the primary reason behind the ineffective security of the Indian Railways?
A. Lack of coordination between the local police and B. Interference of state governments in deploying the
the RPF. police force.
D. The inability of the Railway Police to safeguard
C. Lack of adequate personnel for railway protection
proper defence equipment
E. RPF’s inclination for being over-reliant on local
police.
10. Which of the following is true as per the passage?
A. Railway’s importance is marked by the amount of B. The railway is the easiest target for the
cargo it transports each day. miscreants to attack.
C. There has been an increased threat to railway D. RPF could do better in protecting the railway’s
properties in recent times. assets.
www.byjusexamprep.com

E. Railway is at the forefront of our economic activity.


11. Which of the following can be inferred from the passage?
A. State police forces and GRP are not efficient B. Strengthening the workforce to protect the
enough to protect railway’s assets. railway’s assets is the need of the hour.
C. The Centre needs to nullify the influence of state D. Increased security of railway coaches should be
governments on railway safety measures. prioritised.
E. All of the above
12. Which of the following, if true, would weaken the main argument of the passage?
A. More trains are getting burnt down due to the
B. The government has been unable to set a clear
government’s inability to deploy more security
demarcation of the roles of RPF and local police.
personnel.
C. There have been instances of people getting D. It has been pointed out that local police have
caught in a mix-up between RPF and local police. been derelict in their duties of preserving trains.
E. Railway coaches getting burnt down should not
be made a big deal.
13. Direction: Given below are a set of jumbled up sentences. You are expected to rearrange the sentence to
come up with a meaningful paragraph.

A) A shooting at a Texas elementary school killed 19 children and two adults.

B) It has also been 10 years since the Sandy Hook Elementary School massacre in which 20 children had
died.

C) This was the 27th school shooting in the US this year.

D) But despite a high rate of gun violence, the country’s politicians and voters continue resisting changes
to their gun laws.
A. ABDC B. ACDB
C. ADBC D. ACBD
E. ABCD
14. A) The proof will be in its implementation, but at least it acknowledges the party’s shortcomings.

B) The party plans to give more opportunities to the youth, limit dynastic politics with a ‘one family, one
ticket’ rule,
www.byjusexamprep.com

C) The three-day Nav Sankalp Chintan Shivir of the Congress in Udaipur has charted an action plan for its
revival.

D) and have a new deal for socially weaker groups such as the Scheduled Castes (SC), Scheduled Tribes
(ST), and the minorities.
A. ADBC B. BADC
C. CABD D. DBCA
E. DCAB
15. 1. Meanwhile, all climate indicators continue to break records, forecasting a future of ferocious storms,
floods, droughts, wildfires, and unlivable temperatures in vast swathes of the planet.

2. Nero was famously accused of fiddling while Rome burned.

3. As the fallout of Russia’s invasion of Ukraine ripples across the globe, the response of some nations to
the growing energy crisis has been to double down on fossil fuels, pouring billions more dollars into the
coal, oil and gas that are deepening the climate emergency.

4. Today, some leaders are doing worse. They are throwing fuel on the fire. Literally.

5. Fossil fuels are not the answer, nor will they ever be. We can see the damage we are doing to the
planet and our societies.
A. 23451 B. 24315
C. 21354 D. 24315
E. 23154
16. 1. Souvenirs, an omnipresent facet of modern tourism, trace their roots to the ancient Mediterranean.

2. Indeed, a broad range of souvenirs commemorating places emerges from the archaeological record.

3. Souvenirs then and now offer a remarkable window on how people develop shared visions of places,
how they conceive of such foundational ideas as authenticity, and how we create emotionally meaningful
personal relationships.

4. In the Roman Empire, the common languages of Greek (koine) and Latin, standardised coinage and
centralised bureaucracy increased the ease of travel, all of which helped a culture of souvenirs flourish.
www.byjusexamprep.com

5. These are not just trivial mementoes.


A. 12345 B. 13452
C. 14253 D. 15324
E. 14532
17. Direction: Identify the error in the sentence given below and mark the answer that contains a grammatical
error.

The freelancer had offered a documentation assignment, but he turned it down due to lack of time.
A. The freelancer had offered B. a documentation assignment
C. he turned it down D. due to lack of time
E. No error
18. The students of Kendriya Vidyalaya are expecting a visit from an imminent social scientist.
A. the students of Kendriya Vidyalaya B. are expecting
C. a visit from an D. imminent social scientist
E. No error
19. Regard of whether the students were prepared or not, the instructor declared the dates of the
examination.
A. the students were B. regard of whether
C. prepared or not D. the instructor declared
E. No error
20. The United Nations’ policy regarding environment protection need to become stricter if we wish to battle
climate change in the long run.
A. environment protection B. need to become stricter
C. United Nations’ policy regarding D. battle climate change
E. in the long run
21. Despite of his tiredness, Rohan went to get his bike repaired before the weekend.
A. Rohan went B. despite of his tiredness
C. to get his D. bike repaired
E. before the weekend
22. Direction: In the questions given below, a part of the sentence is italicised and underlined. The
alternatives to the italicised part which may improve the sentence are given below. Choose the correct
www.byjusexamprep.com

alternative.

If it will rain tomorrow, the IPL final will be played the day after tomorrow.
A. If it would rain tomorrow B. If it would have rained tomorrow
C. If it rains tomorrow D. If it is raining tomorrow
E. None of the above
23. Even though I was very young, yet I remember a little about the war.
A. I remember a little about the war. B. but I remember a little about the war.
C. still I was remembering a little about the war. D. still I remember a lot about the war
E. No of the above
24. Direction: Choose the set of prepositions whose meaning and sequence best fits the given three
sentences.

1. The flight from Singapore was going to arrive shortly_______ the airport.

2. Raheem Sterling missed the target _______ 6 yards in the game against Lyon.

3. He was _______ a lot of pressure from his board to resign.

A) to

B) from

C) at

D) despite

E) under

F) into
A. CED B. CBD
C. CAE D. CBE
E. CAB
25. 1. Shane was riding _______ a horse’s back when the accident took place.

2. Riya was not allowed to let anyone _______.


www.byjusexamprep.com

3. He lived on the ground floor, just _______ mine.

A) on

B) from

C) at

D) in

E) with

F) below
A. BED B. ABD
C. ABE D. ADF
E. CAB
26. Direction: Fill in the blanks with the most appropriate preposition from the given answer options.

She had to find a way to prevent investors ______ crippling her logic because she felt that every decision
of hers was being scrutinised and criticised even though she was the CEO of the company.
A. beside B. across
C. to D. from
E. No preposition
27. Direction: Select the most appropriate synonym of the given word.

CAROUSE
A. Mourn B. Overlook
C. Frolic D. Disrespect
E. Demonic
28. Direction: Select the antonym of benediction.
A. Criticism B. Invocation
C. Beatitude D. Consent
E. Compliment
29. Direction: Select the word which is closest in meaning to the given word.
www.byjusexamprep.com

PROBITY
A. Baseness B. Honour
C. Disgrace D. Lowness
E. Inundation
30. Direction: Select the most appropriate antonym of the given word.

ADJOURN
A. Prorogue B. Recess
C. Dangle D. Convene
E. Contravene
31. Direction: In each of the following questions, a related pair or phrases is followed by pairs of words or
phrases. Select the pair that best expresses a relationship similar to that expressed in the original pair.

Inflate:Burst ::
A. Wither:Vanish B. Pull:Tear
C. Expose:Defraud D. Exhume:Increase
E. Wrinkle:Stretch
32. Flippant:Respectful
A. Brusque:Ignorance B. Austere:Steadfastness
C. Droll:Comicalness D. Nonchalant:Concerned
E. Temper:Object
33. Direction: Each sentence below has one or two blanks and each blank indicates that something has been
omitted. Beneath the sentences are some words or phrases. Choose the word or phrase for each blank
that best fits the meaning of the sentence as a whole.

One of the scariest massacres in the history of mankind—Tiananmen Square—still horrifies people.
However, it may not horrify the coming generations, as the history books of China lack any ___ of it.
A. defiance B. documentation
C. conservation D. theory
E. exuberance
34. Gold is used in the semiconductor chips deployed in cellphones. When we dispose off cellphones,
computers, or laptops, we dispose off ___ of gold. A few people are working to make this gold reusable.
www.byjusexamprep.com

However, their works are limited as the methods are not ___.
A. a lot, unprofitable B. accessible, fringe
C. unidentified, subsidised D. plenty, profitable
E. amply, comparable
35. Although many people admired David for his elegance while batting, only a few ___ him for his technical
nuances.
A. praised B. reviled
C. analysed D. criticised
E. summarised
36. Even the elders were so ______________ by the chocolate that they started fighting over it.
A. tempting B. tempt
C. temptation D. tempted
E. tempest
37. Last year, the Indian cricket team played 92 matches. They managed to win only 25% of the matches.
This year, so far, it has played some matches such that the team must win 75% of the remaining matches
to maintain its existing winning percentage. Find the number of matches played by India so far this year.
A. 35 B. 40
C. 43 D. 46
E. Insufficient data
38. In an exam, there are 20 questions worth 130 points. The exam consists of MCQs worth 5 points each
and TITA questions worth 10 points each. How many TITA questions are on the test?
A. 7 B. 5
C. 6 D. 8
E. 14
39. In a PG of 6 boys and a few girls, the average monthly consumption of meal per head is 8 kg. If the
average monthly consumption per head is 10 kg in the case of boys and 7 kg in the case of girls, find the
number of girls in the PG.
A. 15 B. 16
C. 14 D. 12
E. None of these
40. There are eight pairs of yellow sandals and six pairs of blue sandals. These are kept in a bag, and the
www.byjusexamprep.com

sandals are drawn one at a time. To ensure that at least one pair of yellow sandals is taken out, find the
number of sandals required to be drawn out.
A. 19 B. 20
C. 21 D. 22
E. 23
41. Two jeeps start from the same point simultaneously and in the same direction. The first jeep travels at 40
km/h, and the speed of the second jeep is 25% more than the speed of the first jeep. After half an hour, a
third jeep starts from the same point and in the same direction. It overtakes the second jeep 90 minutes
after overtaking the first jeep. What is the speed (in km/h) of the third jeep?
A. 40 B. 45
C. 50 D. 55
E. 60
42. If the roots of the equation differ by 3, then which of the following is true?
A. B.
C. D.
E. None of these
43. Direction: Study the following line graph carefully, and answer the questions given below.

The following line graph shows the speed of different boats in still water in kmph. While going upstream, a
boat goes against the direction of the flow of the river. When the boat goes downstream, it goes in the
same direction as the direction of the flow of the river.
www.byjusexamprep.com

Boat E took 5 hours to cover a distance of 80 kilometres upstream. Find the time that will be taken by Boat
B to cover the same distance downstream?
A. 3 hours B. 4 hours
C. 5 hours D. 6 hours
E. 7.5 hours
44. If the total distance travelled by boats C and D downstream is 300 km in 20 hours, then find the distance
travelled by Boat D downstream if the speed of the stream is 2 km/h.
A. 100 km B. 120 km
C. 140 km D. 160 km
E. 150 km
45. There is another boat, F, and there is a race between boats F, A, and C for 280 km upstream from the
same starting point. Boat F beats Boat A either by 70 km or by 5 hours. Find the distance by which Boat A
beats Boat C in the same race.
A. 100 km B. 110 km
C. 120 km D. 140 km
E. None of the above
46. The speed of the stream on a certain day was 4 km/h. Find the percentage by which the time taken by
Boat B is more/less than the time taken by Boat C if both the boats travelled downstream a distance of
www.byjusexamprep.com

112 km.
A. 10% B. 10.5 %
C. 12% D. 12.5%
E. 15%
47. What will be the remainder when ( ) is divided by 7?
A. 1 B. 0
C. 3 D. 2
E. 5
48. ABC printing company prints 6000 copies of a novel for Rs. 600000 every month. In August, the company
distributed 600 copies for free. Besides, it was able to sell 2/9 of the remaining novels at a discount of
25%. The remaining novels were sold at the printed price of the novel (which was Rs. 240). Find the profit
percentage of the company in the novel business in the month of August. (Assume a uniform 50% of the
sale price as the vendor’s discount. Also, assume that the company earns no income from advertising for
the issue.)
A. 0 B. 1
C. 2 D. 4
E. 5
49. The incomes of A, B, and C are in the ratio of 3 : 5 : 8, and their spendings are in the ratio of 8 : 9 : 10. If A
saves one-third of his income, then what is the ratio of the savings of A, B, and C?
A. 2 : 11 : 24 B. 3 : 9 : 22
C. 3 : 11 : 22 D. 4 : 11 : 22
E. 4 : 9 : 22
50. There were three types of desserts at a party: Kheer, Rasmalai, and Halwa. The total number of people in
the party was 2500, out of which 1450 people ate Kheer and 510 people ate both Kheer and Rasmalai.
The number of people who ate Halwa was found to be 200 more than the number of people who ate
Rasmalai. It was also observed that 120 people ate all three desserts. Find the maximum possible number
of people who did not eat any of the three desserts.
A. 980 B. 990
C. 1030 D. 1050
E. 1120
51. A tank of 4560 cubic metres capacity is being filled with water. The rate of delivery of the pump
www.byjusexamprep.com

discharging the tank is 25% more than the rate of delivery of the pump filling the same tank. As a result, it
takes twelve minutes more to fill the tank than to discharge it. Find the delivery rate of the pump
discharging the tank in litres/minute.
A. 85 B. 90
C. 95 D. 100
E. 105
52. Two integers x and y are chosen such that 3x + 4y = 103. How many pairs of (x, y) exist such that x < 80
and y < 0?
A. 9 B. 10
C. 11 D. 12
E. None of the above
53. Direction: Study the following information carefully to answer the given questions.

The following table shows the data for the users of the four mobile phone brands in different cities:

The integer values denote the number of mobile phone consumers who use the mobile phones of a
particular brand in the given city, whereas the percentages denote the percentage of the users who use
the mobile phones of that brand out of the total mobile phone users in that city. For example, out of the
total mobile phone users in Delhi, 26% of the consumers use the mobile phones of Viva.

Note: Number of mobile phones sold by a brand in a city = Number of mobile phone users in that city of
that brand

How many cities are there in which at least 750 mobile phone users of each brand are there?
A. 1 B. 2
C. 3 D. 4
E. 5
54. How many mobile phones are sold by Viva in these five cities?
www.byjusexamprep.com

A. 9050 B. 9100
C. 9155 D. 9055
E. None of the above
55. What is the ratio between the total number of mobile phone users in Delhi, Mumbai, and Pune?
A. 3 : 2 :5 B. 8 : 12 : 5
C. 7 : 10 : 9 D. 6 : 11 : 13
E. 9: 12 : 5
56. What is the average number of users of these four brands in the five cities taken together?
A. 5000 B. 5200
C. 5400 D. 5600
E. 6000
57. A college has three departments P, Q, and R, and the population of the students in these departments are
in the ratio of 4 : 5 : 10. In department P, 75% of the students are females, and in department Q, 40% of
the students are males. If 80% of the students in department R are females, find the approximate
percentage of females in the college.
A. 58% B. 65%
C. 68% D. 70%
E. 74%
58. Find the number of ways in which 360 can be expressed as a product of two factors.
A. 20 B. 12
C. 10 D. 8
E. 5
59. The length and speed of Train A are 300 m and 36 km/h, respectively. The length and speed of Train B
are 500 m and 72 km/h, respectively. How long (in seconds) will it take a person in Train B to overtake
Train A?
A. 10 B. 15
C. 20 D. 25
E. 30
60. If f(x2 – 9x + 12) = 5x + 9, find f(–6).
A. 21 B. 24
C. 39 D. 40
www.byjusexamprep.com

E. Both B and C
61. Three types of premium rice brands, A, B, and C, cost Rs. 210/kg, Rs. 230/kg, and Rs. 320/kg,
respectively. In which of the following ratios should the three types of rice be mixed such that the cost of
the mixture is Rs. 270?
A. 1 : 3 : 2 B. 15 : 5 : 21
C. 5 : 15 : 18 D. 2 : 3 : 4
E. None of these
62. Direction: Study the following information carefully, and answer the question given below.

The following bar graph shows the percentage marks obtained by four students in three different subjects:
mathematics, physics, and chemistry. The maximum marks for each subject is given in the graph.

Maths in the graph refers to mathematics.

What is the sum of the average marks obtained by all four students in the three subjects given?
A. 316 B. 326
C. 416 D. 426
E. 336
63. What is the ratio of the sum of marks obtained by Sanjay in mathematics and physics to the sum of marks
obtained by Nitin in physics and chemistry?
A. 20 : 13 B. 25 : 12
www.byjusexamprep.com

C. 15 : 11 D. 11 : 15
E. None of the above
64. Let us assume that the marks in theory exams of physics and chemistry are 70% and 80%, respectively,
and the remaining marks are considered for practical exams. How many marks does Arun get in English if
the marks obtained by Arun in English is 25% more than the sum of marks obtained by Varun and Sanjay
in practical exams of Physics and Chemistry, respectively?
A. 50 B. 55
C. 60 D. 70
E. 75
65. If another student Vijay scored 20% more marks in mathematics, 25% less marks in physics, and same
marks in chemistry as compared to Arun, then what is the approximate percentage difference in marks
obtained by Vijay and Sanjay in the three subjects taken together?

[Note: For approximate percentage value of the percentage marks, neglect the decimal part.]
A. 1% B. 5%
C. 3% D. 4%
E. 2%
66. What is the probability that a random year has 53 Saturdays?
A. B.

C. D.
E. None of these
67. A shopkeeper marked the price of an item 80% above the cost price. He allowed two successive
discounts of 25% and 33.33% to a customer. If he incurred a loss of Rs. 320, at what price did he sell the
item to the customer?
A. Rs. 3000 B. Rs. 3200
C. Rs. 2870 D. Rs. 2880
E. None of these
68. If log (x2 – 11x + 25) = 0, then which of the following can be the value of log (9x)?
7 x

A. 2 B. log89 + 1
C. 2log98 D. 4
www.byjusexamprep.com

E. 3log98
69. Akarsh invested a certain sum of money in a simple interest bond whose value increased by Rs. 900 at
the end of 3 years and Rs. 1500 at the end of 8 years. What was the rate of interest in which he invested
his sum?
A. 22.22% B. 18.18%
C. 21.21% D. 16.67%
E. None of these
70. Two pipes A and B together can fill a tank in 24 minutes. On a particular day, Pipe A is opened in an
empty tank, and as soon as the tank is half full, it is closed and Pipe B is opened. Thus, the tank is filled in
a total of 75 minutes. Now, if there is a leak because of which Pipe A takes 40 minutes to fill the tank, then
how much time will Pipe B alone take to fill the tank?
A. 1 hours B. 1.5 hours
C. 2 hours D. 2.5 hours
E. Cannot be determined
71. Direction: The question given below is followed by two statements, I and II. You have to determine
whether the data given in the statements are sufficient to answer the question and you have to choose the
best possible answer.

Find the minimum possible number of factors of N.

Statement I: N has at least 4 distinct prime factors.

Statement II: N is divisible exactly once by all its prime factors.


A. The data given in Statement I alone is sufficient to B. The data given in Statement II alone is sufficient
answer the question, while the data given in to answer the question, while the data given in
Statement II alone is not sufficient to answer the Statement I alone is not sufficient to answer the
question. question.
C. The data given in either Statement I or Statement D. The data given in both the statements together is
II alone is sufficient to answer the question. required to answer the question.
E. The data given in both the statements together is
not sufficient to answer the question.
72. Direction: The question given below is followed by two statements, I and II. You have to determine
whether the data given in the statements are sufficient to answer the question, and you have to choose
www.byjusexamprep.com

the best possible answer.

Find the ratio of profit earned by C at the end of the year.

Statement I: The ratio of the amounts in which A, B, and C invested in the business is 2 : 3 : 4

Statement II: The ratio of the durations for which A, B, and C invested in the business is 4 : 3 : 2
A. The data given in Statement I alone is sufficient to B. The data given in Statement II alone is sufficient
answer the question, while the data given in to answer the question, while the data given in
Statement II alone is not sufficient to answer the Statement I alone is not sufficient to answer the
question. question.
C. The data given in either Statement I or Statement D. The data given in both the statements together is
II alone is sufficient to answer the question. required to answer the question.
E. The data given in both the statements together is
not sufficient to answer the question.
73. Which of the following numbers will replace the question mark?

A. 10 B. 9
C. 8 D. 16
E. 7
74. Which of the following numbers will replace the question mark?
www.byjusexamprep.com

A. 5 B. 6
C. 8 D. 9
E. None of the above
75. Direction: Given below is a statement followed by two courses of action, I and II. You are required to go
through the statement and check whether the given courses of action follow or not.

Statement: Ankur wants to switch his field of work, but he is not sure where to start.

Courses of action:

I. Ankur first needs to figure out which field of study he is going to pursue.

II. Ankur needs to research a lot about the change of streams before taking a decision.
A. Only I follows. B. Only II follows.
C. Both I and II follow. D. None follows.
E. Either I or II follows.
76. Statement: Heavy rain has disrupted normal life in several parts of the country, especially the
northeastern states. Situation remains grim in flood-hit parts of Assam, leaving over millions affected.

Courses of action:

I. If the situation worsens, the army must be deployed at the earliest to take on rescue operations.

II. The people living in low-lying areas must be evacuated immediately.


A. Only I follows. B. Only II follows.
C. Both I and II follow. D. None follows.
www.byjusexamprep.com

E. Either I or II follows.
77. Statement: The Reserve Bank will further hike benchmark interest rates, and the spending squeeze that
would follow will likely slow down the economy, members of the central bank’s Monetary Policy Committee
noted in its last meeting.

Courses of action:

I. The Reserve Bank has no right to hike benchmark interest rates, given the poor state of the economy.

II. The economic slowdown should be handled with deft handling on inflation via incentive schemes for
small and large businesses to drive growth.
A. Only I follows. B. Only II follows.
C. Both I and II follow. D. None follows.
E. Either I or II follows.
78. Statement: Drivers around the world are feeling pain at the pump with fuel prices soaring, and costs are
surging for heating buildings, power generation, and industrial production.

Courses of action:

I. The governments around the world should increase subsidies on fuel prices to keep them under check.

II. The Russia-Ukraine war must be stopped at all costs for improved business relations around the world.
A. Only I follows. B. Only II follows.
C. Both I and II follow. D. None follows.
E. Either I or II follows.
79. Direction: Study the following information carefully, and answer the questions given below.

When given an input line of letter clusters, a word arrangement machine rearranges them by following a
particular rule in each step. The following is an illustration of the input and the rearrangement:

Input: VERT BHUX AEDT NKJL LVRS


Step I: XGTV DJWZ CGFV PMLN NXTU
Step II: GTVX DJWZ CFGV LMNP NTUX
Step III: TGEC WQDA XUTE ONMK MGFC
Step IV: 12 21 41 27 13
www.byjusexamprep.com

Step V: C C E I D

Step V is the last step of the rearrangement. As per the pattern followed in the above steps, find the
appropriate steps for the following input and then answer the questions.

Input: CDFV BGHN MJKL QWER TAUI

What is the smallest number in Step IV?


A. 35 B. 40
C. 27 D. 21
E. None of these
80. Which element is second from the right end in Step II?
A. CKVW B. GSTY
C. LMNO D. DIJP
E. None of these
81. What is the sum of Step IV's first and last elements?
A. 55 B. 61
C. 67 D. 63
E. None of the above
82. If the words in Step II are arranged alphabetically from right to left, then which of the following would
represent the new Step II?
A. CKVW DIJP EFHX GSTY LMNO B. EFHX DIJP GSTY LMNO CKVW
C. GSTY CKVW EFHX LMNO DIJP D. EFHX CKVW GSTY LMNO CKVW
E. None of the above
83. Direction: In the question below, a statement is given followed by two arguments. Choose the most
appropriate option that tells us whether that particular argument is strong or not.

Statement: India’s greatest national treasure is its people—especially women and children—but even
after 75 years of independence, a majority of them do not get the required diet to meet their nutritional
needs. So, should the government intervene to make things better for the underprivileged section of the
society?
www.byjusexamprep.com

Arguments:

I. Yes, because if the government does not help them out, who will do the same?

II. Yes, because a country’s comprehensive development lies in its ability to provide its underprivileged
sections the basic necessities.
A. Only argument I is strong. B. Only argument II is strong.
C. Both arguments I and II are strong. D. Neither argument I nor II is strong.
E. Either I or II is strong.
84. Statement: On the occasion of International Museum Day, the Ministry of Culture has made admissions to
all museums under its ambit free of cost for a week. So, is this a good step?

Argument:

I. Yes, because it not only makes our art and culture accessible to all, but also provides us with an
opportunity to propagate our cultural heritage.

II. No, because it would lead to a huge loss of money from gate receipts.
A. Only argument I is strong. B. Only argument II is strong.
C. Both arguments I and II are strong. D. Neither argument I nor II is strong.
E. Either I or II is strong.
85. Direction: Study the following information carefully, and answer the questions given below.

When an input of numbers in sequence is given, a number arrangement machine rearranges them by
following a particular rule in each step. The following is an illustration of the input and rearrangement.

Input: 8743 9617 7381 3286 4511 7843


Step I: 3741 1632 5913 2752 495 3354
Step II: 1473 2361 3195 2572 594 4533
Step III: 15 12 18 16 18 15
Step IV: 22 5 25 9 25 8
Step V: 506 30 650 90 650 72
Step V is the last step of the rearrangement.
www.byjusexamprep.com

As per the pattern followed in the above steps, find the appropriate steps for the following input and then
answer the questions.

Input: 7453 2354 7492 6484 9673 4535

Which of the following numbers are exactly in the middle of Step III?
A. 21, 20 B. 19, 9
C. 23, 21 D. 9, 22
E. 22, 21
86. Which of the following is Step II of the arrangement?
A. 2343 2436 6328 6373 8006 5751 B. 2436 7935 7359 7353 7373 9464
C. 2276 2451 8397 6274 8364 6378 D. 2293 2421 8086 6735 8007 5751
E. 2293 2421 8086 6735 8007 6378
87. What is the sum of all the numbers in Step IV?
A. 537 B. 110
C. 133 D. 89
E. 101
88. What is the positive difference between the largest and the smallest numbers in Step V?
A. 451 B. 129
C. 864 D. 678
E. 684
89. Direction: In the question below are given three statements followed by the conclusions. Read all the
conclusions and then decide which of the given conclusions logically follow from the given statements.

Statements:

All cards are posts.

Some posts are not pins.

No pin is a pay.

Conclusions:
www.byjusexamprep.com

I. No pay is a post.

II. Some cards are not pins.


A. Only I follows. B. Only II follows.
C. Both I and II follow. D. Neither I nor II follows.
E. Either I or II follows.
90. Read the following passage and answer the given question:

Signs of confrontation between the Raj Bhavan and an elected state government are not infrequent in the
country.

Which of the following can be inferred from the statement?


B. Differences of opinion between the state and the
A. The state and the central governments are always
central governments are quite common in our
at war with one another.
country.
C. The state and the central governments are bound D. The confrontation between states and the Centre
to lock horns at some stage. is not something to worry about.
E. No amount of friction between the state and the
central government should hamper the progress of
the nation.
91. Read the following passage and answer the given question:

World football’s governing body FIFA ejecting all Russian teams, national representatives or club sides
from its competitions until further notice is the most severe of sporting sanctions imposed in the backdrop
of the Russia-Ukraine crisis.

Which of the following can be inferred from the given information?


A. Any country which invades another country will be B. Imposing a ban on Russia’s footballing teams is
barred from participating in FIFA competitions until FIFA’s way of condemning the war it has waged on
further notice. Ukraine.
C. Russia must be prepared for the grave
D. FIFA is doing the right thing to stop Russia from
consequences of invading Ukraine in the coming
entering FIFA tournaments.
days and months.
E. Both B and D
www.byjusexamprep.com

92. Read the following passage and answer the given question:

The National Power Corporation has been trying to reduce harmful emissions in line with the government’s
target of zero emission by 2050. It has decided to switch to cleaner fuels and also to install cleaning
mechanisms in smokestacks. We can conclude that the company has successfully managed to reduce its
emissions to the maximum possible level by installing cleaning mechanisms in all its major power plants.

Which of the following assumptions supports the argument in the passage?


A. The company found the switch to a cleaner fuel B. The National Power Corporation is a pioneer in
more cumbersome. green cleaning mechanisms in the country.
D. The National Power Corporation cannot further
C. The corporation is not really serious about the
reduce emissions by using both clean fuel and
environment.
cleaning mechanisms.
E. The national power corporation has its hands tied.
93. Read the following passage and answer the given question:

Recent studies reveal rather disconcerting facts about the adverse effects of computer games. The
studies revealed that children who play computer games for long hours in a day have impaired
communication skills. The children play computer games instead of talking to friends and spending time
with them. Therefore, researchers concluded that children who spend all their free time playing computer
games have less interaction with people, and hence, do not have as much experience in interpersonal
communication as non-players have.

Which of the following correctly states the assumption of the argument?


B. It depends on a child’s inherent nature whether
A. Watching television is more passive than playing
they spend time with other children or playing
computer games, and its effects are worse.
computer games.
C. Children who do not spend their free time playing
D. Educational institutions do not help in building
computer games, spend time interacting with other
interpersonal skills.
people.
E. Children should be dissuaded from playing video
games during their free time.
94. Read the following passage and answer the given question:
www.byjusexamprep.com

During the last budget, the government cut defence expenditure. Defence ministry officials complained
that the cuts would compel them to reduce expenditure for essential services and eventually affect
national security. But the defence officials' fear was unfounded as only non-essential expenditure was cut.
So, we can cut defence expenditure further this year without compromising on essential services and
national security or the safety of our defence personnel.

Which of the following, if true, strengthens the argument the most?


A. The defence ministry has sufficient funds for B. The defence ministry has always got a bigger
some non-essential expenditures. share of the budget than it deserved.
D. The government always overlooks the defence
C. The cost of non-essential services has not
ministry’s objections, often leading to disastrous
increased since the last budget.
outcomes.
E. The defence ministry wants to cut corners in order
to have some extra money for developmental
schemes.
95. Direction: Study the following information carefully, and answer the given questions.

Eight persons (F, G, H, I, J, K, L, and M) are sitting in a straight line but not necessarily in the same order.
Some of them are facing north, whereas some are facing south. None of them are facing some other
direction. They like different fruits such as apples, mangoes, kiwis, bananas, papayas, pears, grapes, and
litchis but not necessarily in the same order. They are also born in different months, i.e., January, March,
April, May, June, July, August, and September but not necessarily in the same order.

The one who likes kiwis sits second to the left of the G, who was born in August. Only three people sit
between the one who likes kiwis and the one who likes mangoes. Immediate neighbours of the one who
likes mangoes face opposite directions.

J likes kiwis and was born in June. Only three persons sit between the one who likes kiwis and K, who sits
second to the right of the one who is born in May. The number of people who are sitting to the left of the
one who likes pears is one less than the number of persons who are sitting to the right of E

F who likes bananas is to the immediate right of H. The one who likes papayas does not sit at the extreme
ends of the line. The one who likes litchis sits second from one of the ends of the line and was also born in
January.
www.byjusexamprep.com

The one who likes papayas sits third to the left of J. J and the one who likes litchis are not immediate
neighbours of each other. E does not face south. The one who was born in March sits exactly between G
and K.

G does not like litchis. Immediate neighbours of J face the opposite direction to that of J. If J faces north,
then his immediate neighbours will face south and vice versa. L likes grapes and does not sit at the
extreme ends of the line. Persons who are born in months following one another are not immediate
neighbours of each other.

Who amongst the following sits at the extreme ends of the line?
A. G and the one who likes litchis B. J and G
C. F and the one who likes apples D. I and the one who was born in June
E. Cannot be determined
96. Which of the following is true with respect to K?
A. K faces north. B. K faces south.
C. K faces either north or south. D. K faces east.
E. None of these are true.
97. Which of the following statements is/are incorrect with respect to the given arrangement?
A. I, who likes apples, does not sit at the extreme B. J is an immediate neighbour of the one who likes
ends of the line. grapes.
C. K faces either north or south. D. E sits third to the left of the one who likes kiwis.
E. All the above statements are correct.
98. Who amongst the following sits third to the right of the one who is third to the left of the one who likes
pears?
A. F B. The one who likes pears
C. E D. The one who likes apples
E. None of the above
99. Read the following passage and answer the given question:

A famous pharmaceutical company stated the following in its advertisement: For migraines, seven out of
ten hospitals sell Tablet X. A survey found that 705 patients who suffer from migraine admitted to taking
Tablet X. So, if you want an effective cure for migraine, use Tablet X.
www.byjusexamprep.com

Which of the following, if true, weakens the argument in the advertisement?


B. Clinical trials prove Tablet X to be safe and
A. Tablet Y developed by the pharma’s rival can cure
effective for all kinds of headaches, including
other pains besides migraine.
migraines.
D. Tablet X is an over-the-counter medicine, while
C. Medical representatives are the best
many other migraine medicines cannot be sold
ambassadors for pharma brands.
without a doctor's prescription.
E. Tablet Y needs some alterations in its formula
before it can be used for migraines.
100. Read the following passage and answer the given question:

Acetaminophen or paracetamol helps to reduce fever and relieve a headache with mild to moderate pain.
It is also used with other pain medications to treat low back pain, cold, and toothache. Paracetamol is a
schedule K drug, which means that people can buy it without a prescription. But doctors now are wary of
its side effects and no longer routinely recommend its use. Research has found that prolonged use of
paracetamol can have many adverse effects. Mild fever is the symptom of many illnesses. A mild fever
results in activating the body's natural disease-fighting mechanisms through the production of white blood
cells. Paracetamol prevents even mild fever.

Which of the following conclusions can be drawn from the above passage?
A. Paracetamol can be used alone or in combination B. Paracetamol has many life-threatening side
with other pain medications. effects.
C. Paracetamol can make an illness last longer as it D. Modern medicine has done more harm than good
prevents even a mild fever. with the use of paracetamol.
E. Both B and D
101. Read the following passage and answer the given question:

Banks now have access to signature-recognition software by which they can analyse the cheques of their
customers. The software identifies a person’s signature by analysing not only the form of the signature but
also such characteristics as pen pressure and signing speed. Even the most adept forgers will not be able
to duplicate all the characteristics the software analyses.

Which of the following can be logically concluded from the passage given above?
A. The time it takes to record and analyse a B. Computers equipped with the software will soon
www.byjusexamprep.com

signature makes it impractical for regular use. be installed in most banks.


D. In many cases, even genuine customers are
C. Almost no one will be able to successfully forge
denied access to withdrawing money because of this
signatures in cheques and withdraw money.
software.
E. Forgery of signatures will require a tremendous
amount of work.
102. Read the following passage and answer the given question:

Taking antibiotics can successfully protect you against bacterial infections—and it’s often an important part
of maintaining your health. But where regular consumption of antibiotics was once thought sufficient to
protect you, new research suggests that unhealthy eating habits can reduce the efficacy of antibiotics.

Which of the following can be inferred from the information provided in the question?
B. Our eating habits have a lot to do with how
A. Bacteria have become more averse to antibiotics.
healthy we are.
C. We should stop eating junk food to avoid risks of D. Bacterial infections have also grown over the
bacterial infections. years, nullifying the effects of antibiotics.
E. All of the above
103. Amar, Simar, Bhramar, and Shivar are four players who go for badminton coaching classes with coach
Shivanna. The classes are scheduled for one hour each from 6 a.m. to 10 a.m.. None of the four friends
go to the class at the same time. They use rackets of brands W, X, Y, and Z, not necessarily in the same
order.

Bhramar does not use rackets of brands Y and Z, and he is the first one to go to the class. Shivar is the
last one to go to the class, and he uses a racket of Brand X. The player who goes to class from 8 a.m. to 9
a.m. uses a racket of Brand Z. Simar uses a racket of Brand Y. Which of the following uses a racket of
Brand Z?
A. Amar B. Simar
C. Bhramar D. Shivar
E. Cannot be determined
104. In a certain code language, ‘BOWL’ is written as ‘FRYM’ and ‘READ’ is coded as ‘VHCE’. Which of the
following words is written as LRPL in that language?
A. MONK B. HONK
www.byjusexamprep.com

C. TOMB D. JOMB
E. HUMP
105. Which of the following diagrams best represents the relationship between the given elements?
Prison, Pigeon, Prism

A. B.

C. D.

E.

106. Direction: In each of the questions below are given a few statements followed by two conclusions. You
have to take the given statements to be true even if they seem to be at variance from commonly known
facts. Read all the conclusions and then decide which of the given conclusions logically follow from the
given statements disregarding commonly known facts.

Statements:
Some tigers are panthers.
Some panthers are kangaroos.
All kangaroos are foxes.
Conclusions:
I. All tigers are kangaroos.
II. Only a few tigers being foxes is a possibility.
A. Neither conclusion I nor conclusion II follows. B. If conclusion I follows
C. Both the conclusions follow. D. If conclusion II follows
E. Either Conclusion I or II follows
107. Direction: In each of the following questions, two statements are given which may have a cause and effect
www.byjusexamprep.com

relationship. Identify if the sentences have a causal relationship and mark your answers according to the
given code.

Statements:

I. The Union Ministry has decided to cut fuel costs in the wake of rising inflation.

II. The global price of crude oil has diminished around the world.

Mark:
A. if I is the cause and II is the effect B. if II is the cause and I is the effect
C. if both are independent causes D. if both are independent effects
E. if the statements are not true
108. Direction: In each of the following questions, two statements are given which may have a cause and effect
relationship. Identify if the sentences have a causal relationship and mark your answers according to the
given code.

Statements:

I. The Reserve Bank has decided to reduce interest rates, thereby reducing the cost of borrowing funds.

II. Most small and medium enterprises are on a hiring mode.

Mark:
A. if I is the cause and II is the effect B. if II is the cause and I is the effect
C. if both are independent causes D. if both are independent effects
E. if the statements are not true
www.byjusexamprep.com

Solutions

1. B
Sol.

Passage summary: This article talks about one of the most dangerous female spies, Virginia
Hall, and her work during World War II.

Genre: History

Number of words: 499

Type of question: Specific idea-based question

In the fifth paragraph, it is mentioned that Virginia volunteered in France as an ambulance


driver. Refer to this line: ‘Frustrated, she resigned from her administrative job at the U.S.
embassy in Estonia and volunteered in France as an ambulance driver in 1940 …’

Thus, Statement II is correct.

In the fourth paragraph, it is mentioned that Virginia Hall worked in various clerical roles. Refer
to this line: ‘Despite gaining experience in various clerk roles at consulates across Europe …’

Thus, Statement IV is correct.

Statement I is incorrect. Virginia worked as a spy agent in France under the guise of a reporter
with the New York Post named Brigitte LeContre based in Lyon.

Virginia wanted to work as a diplomat in the State Department, but was rejected for the role.
Refer to this line: ‘… Hall’s multiple attempts to become a diplomat in her own right with the
State Department were all rejected …’ Thus, Statement III is incorrect.
www.byjusexamprep.com

Thus, only statements II and IV are correct.

Hence, option B is the correct answer.

2. C
Sol.
Passage summary: This article talks about one of the most dangerous female spies, Virginia
Hall, and her work during World War II.

Genre: History

Number of words: 499

Type of question: Direct/Fact-based question

The sixth and seventh paragraphs of the passage explain the work carried out by Virginia in
France as a spy agent in detail.

Option A mentions one of the works carried out by Virginia, as mentioned in these lines from
the passage: ‘Hall also planned jailbreaks for other agents who had been captured, and in all,
survived alone in the field for 12 months before other women spies were deployed to France.’

Option B also mentions the work of Virginia. Refer to this line: ‘Hall’s activities in galvanising
the Resistance were successful, and had attracted the attention of the French Vichy Police
and the Gestapo.’

Option C is not mentioned in the passage as work done by Virginia. Thus, option C is correct.

Option D is the work of Virginia.

Refer to this line: ‘Her status as a journalist and her charming manner meant that she quickly
established a network of contacts and was able to recruit more people to the resistance.’

Option E is stated in the seventh paragraph.


www.byjusexamprep.com

Hence, the correct answer is option C.

3. B
Sol.
Passage summary: This article talks about one of the most dangerous women spies, Virginia
Hall, and her work during World War II.

Genre: History

Number of words: 499

Type of question: Direct/ Fact based Question

This is a direct question. She was rejected because she was a woman and disabled. Refer to
this line: ‘Hall’s multiple attempts to become a diplomat in her own right with the State
Department were all rejected on account of her being both a woman and disabled.’

Hence, the correct answer is option B.

4. D
Sol. Passage summary: This article talks about one of the most dangerous female spies, Virginia Hall, and her
work during World War II.

Genre: History

Number of words: 499

Type of question: Specific idea-based

Option A is true and is mentioned in the last line of the passage.

Option B is also correct. It is clear from these lines from the passage: ‘Under the codename “Germaine,”
Hall also planned jailbreaks for other agents who had been captured, and in all…’

Also, note this line: ‘… but due to her adept use of disguises, the authorities never quite worked out
exactly who “Germaine” was ...’
www.byjusexamprep.com

Option C is true. Refer to this line: ‘…recounting how a young Virginia was ambitious and set her sights on
studying in Paris after attending George Washington University.’

Option D is incorrect. Virginia was left amputated because she had accidentally shot herself in the foot on
a hunting trip in Turkey.

Option E is correct and is stated in the third paragraph.

Hence, option D is the correct answer.


5. B
Sol.
Passage summary: The article discusses global carbon emission numbers surrounding food
transportation and production, as the author cites actual numbers for a comparative analysis
of the emissions food products are responsible for.

Number of words: 402

Genre: Economy/Ecology

Type of question: Specific idea question

Refer to these lines:

‘Transporting food from where it is produced to our dinner plates creates at least triple the
amount of greenhouse gas emissions as previously estimated, a new study suggests. So-
called food miles are likely responsible for about 6% of the world’s greenhouse gas
emissions…’

From the context, it is clear that the term food miles refers to the number of miles food has to
travel to reach consumers. Hence, it is called food miles, and hence, option B is the correct
answer.

Options A and C are not true according to the extract given above.
www.byjusexamprep.com

Option D is rejected because food’s entire lifetime does not make any sense here. If we are
considering food’s entire lifetime, then it can also include those food items which are spoiled
before reaching the consumer. Option E is written from the perspective of retailers, which is
irrelevant here.

Hence, the correct answer is option B.

6. C
Sol.
Passage summary: The article discusses global carbon emission numbers surrounding food
transportation and production as the author cites actual numbers for a comparative analysis of
the emissions food products are responsible for.

Number of words: 402

Genre: Economy/Ecology

Type of question: Inference question

First of all, let’s look at the numbers surrounding richer nations and food miles emissions:

‘Scientists who carried out the research, which analysed 74 countries and regions and 37
different types of food, found richer countries accounted for 46% of the emissions from food
miles but just 12.5% of the world’s population.’

Given that richer nations account for 46% of food miles emissions while serving only 12.5% of
the world's population, it is clear that richer nations have hugely contributed to food miles
emissions. Hence, option C is the correct answer.

Option D is rejected because it talks about the global climate crisis as a whole, thereby
widening the scope of the passage.

Option E is not true in the given context.


www.byjusexamprep.com

Options A and B are irrelevant.

Hence, the correct answer is option C.

7. A
Sol.
Passage summary: The article discusses global carbon emission numbers surrounding food
transportation and production as the author cites actual numbers for a comparative analysis of
the emissions food products are responsible for.

Number of words: 402

Genre: Economy/ Ecology

Type of question: Main idea question

Throughout the passage, the author has primarily talked about the numbers surrounding
global carbon emissions in terms of food transportation and production. Using the same, the
author has tried to demonstrate that Food miles have affected climate change more than what
was perceived. The same can be corroborated by the given statements:

‘Transporting food from where it is produced to our dinner plates creates at least triple the
amount of greenhouse gas emissions as previously estimated, a new study suggests.’

Hence, option A is the correct answer.

8. B
Sol.
Passage summary: The article discusses global carbon emission numbers surrounding food
transportation and production as the author cites actual numbers for a comparative analysis of
the emissions food products are responsible for.

Number of words: 402

Genre: Economy/Ecology
www.byjusexamprep.com

Type of question: Specific idea question

Refer to these lines:

‘Fruit and vegetables had the highest food miles emissions because they often needed to be
refrigerated and consumers demanded out-of-season foods.’

Now, since fruits and vegetables have the highest food miles due to their out-of-season
demand, if people’s penchant for out-of-season fruits and vegetables diminishes significantly,
then it would seriously dent the number of carbon emissions from food miles.

Hence, option B is the correct answer.

9. A
Sol.
Passage summary: The article talks about the damages incurred to railway assets in the wake
of the Agneepath scheme protests as the author highlights why trains are an easy target for
demonstrators and what can be done to improve the status quo.

Genre: Current affairs

Number of words: 340

Type of question: Specific idea question

Refer to the following lines from the passage:

‘RPF does not lack attention. It is not only empowered to arrest without a magistrate’s order, in
2019 handpicked personnel were given commando training. If IR’s security performance is still
unsatisfactory, it is because of gaps in coordination with the state police.’

Here, the author has clearly highlighted the lack of coordination between the state police and
the railway police as the reason behind the ineffective protection of railways. Hence, option A
is the correct answer.
www.byjusexamprep.com

10. A
Sol.
Passage summary: The article talks about the damages incurred to railway assets in the wake
of the Agneepath scheme protests as the author highlights why trains are an easy target for
demonstrators and what can be done to improve the status quo.

Genre: Current affairs

Number of words: 340

Type of question: Conclusion-based question

It is clearly mentioned that not only does the railway’s destruction cause inconvenience to a lot
of people, but it also disrupts the economy because it carries over 3.3 million tons of freight
daily, mainly coal. Hence, we can say that a railway’s importance is marked by the amount of
cargo it carries each day. Hence, option A is the correct answer.

11. A
Sol.
Passage summary: The article talks about the damages incurred to railway assets in the wake
of the Agneepath scheme protests as the author highlights why trains are an easy target for
demonstrators and what can be done to improve the status quo.

Genre: Current affairs

Number of words: 340

Type of question: Inference-based question

Refer to the following line:

‘GRP and state police forces handle the security of trains, tracks and railway premises, while
RPF protects properties and consignments. The performance of local policing is influenced by
www.byjusexamprep.com

a state government’s political signalling. Consequently, the leeway agitations have a direct
impact on IR’s assets.’

Here, focus on the word ‘consequently’, which clearly suggests that GRP and state police
handling the security of trains results in agitations getting out of hand and, hence, affecting the
security of railway’s assets. Hence, we can say that option A is the correct answer.

12. A
Sol.
Passage summary: The article talks about the damages incurred to railway assets in the wake
of the Agneepath scheme protests as the author highlights why trains are an easy target for
demonstrators and what can be done to improve the status quo.

Genre: Current affairs

Number of words: 340

Type of question: CR-based question

To answer this question, you first need to know the main idea of the passage. The passage
has clearly mentioned that the reason behind railway properties getting damaged is improper
coordination between RPF and local police. Now, to weaken this statement, we need to make
sure we pick an answer option that shifts the blame entirely on the government because if it
manages to do so, then the whole point of no coordination between the two police forces
would not matter. Here, option A does the same because it puts the blame on the
government’s inability to deploy more security personnel. Hence, option A is the correct
answer.

13. D
Sol. Statement A opens the paragraph by talking about a shooting in a school that killed 19
children. Now, C-B forms a logical link that connects this years’ unfortunate events to a
www.byjusexamprep.com

particular event that was first in line and took place 10 years ago. So, ACB forms the link,
which is then completed by D.

Hence, option D is the correct answer.

14. C
Sol.
Here, the key to the answer lies in figuring out the opening statement, which is nothing but the
most generic statement of the lot. Here, statement C fulfils the criteria as it introduces us to the
idea of the Congress Party holding a Chintan Shivir in Udaipur. Now, statement A follows
statement C as it talks about what the Shivir represents. Statements B and D are obviously
linked by the conjunction ‘and’. Hence, the correct answer is option C.

15. B
Sol. Genre: Ecology

First of all, we need to keep in mind that the author has opened the paragraph with an
analogy. Just like what Nero did during the burning of the Roman Empire was not acceptable,
today’s leaders taking a similar step in the wake of the growing climate crisis is also not
acceptable. So, 2-4 is clearly a mandatory pair. Now, 4 should be followed by 3 because it
introduces us to the issue of global warming, which has been made worse in the wake of the
Russia-Ukraine war. Also, 3 should be followed by 1 because it talks about the worsening of
the situation as a result of what has occurred due to statement 3. Thus, 24315 is the right
answer here.

Hence, the correct answer is option B.

16. C
Sol. Genre: History/Anthropology

First of all, the opening statement is talking about souvenirs and the author has linked their
origin to the ancient Mediterranean. Now, in order to find the next statement, we first need to
know which of the given statements establishes the presence of souvenirs in the ancient
Mediterranean.
www.byjusexamprep.com

Now, Mediterranean countries are found in Europe and hence, we’ll need to follow statement 1
with 4 because it talks about the presence of souvenirs in these countries during the Roman
Empire. After 4, 2 should follow because of the word ‘indeed’ as it is used to add information to
a given statement. Hence, 4-2 is a mandatory pair and 4-2 is also the opening pair.

Hence, option C is the correct answer.

17. A
Sol.
Option A has the grammatically incorrect part. A passive voice should be used here. When we
want to focus on the person or thing affected by the action, we make the person or thing the
subject of the sentence and use the passive voice. Hence, replace ‘he had offered’ by ‘he had
been’.

Hence, option A is the correct answer.

18. D
Sol.
Eminent is an adjective that means famous and respected within a particular sphere and
generally refers to a person. Alternatively, it may also mean present to a notable degree.
Imminent refers to something that is about to happen. The synonyms of imminent are looming,
impending, forthcoming, and more. The error is in part D.

Hence, the correct answer is option D.

19. B
Sol.
Regard means to consider or think about something in a particular way, or to look carefully at
someone or something. Regardless means not being affected by something, despite the
prevailing circumstances. It should be regardless in place of regard.

Hence, option B is the correct answer.


www.byjusexamprep.com

20. B
Sol.
Here, the error is related to the verb. Here, the word need is a verb (and not a modal). The
verb should agree with the subject. The subject ‘United Nations policy regarding environment
protection’ is a singular subject. Thus, need should be replaced with needs to make the
sentence grammatically correct.

Hence, the correct answer is option B.

21. B
Sol.
The error is in the usage of the preposition of. We do not use of after despite. Despite is a
preposition which means without taking any notice of or being influenced by; not prevented by.
Thus, remove of to make the sentence grammatically correct.

Hence, the correct answer is option B.

22. C
Sol. The given sentence is a type I conditional sentence. The correct structure is present.

If present tense, then simple future.

If it rains, the final will be played later.

Hence, the correct answer is option C.


23. A
Sol. We do not use yet with though or even though.

Hence, the correct answer is option A.


24. D
Sol.
The correct sentences are:

1. The flight from Singapore was going to arrive shortly at the airport.
www.byjusexamprep.com

2. Raheem Sterling missed the target from 6 yards in the game against Lyon.

3. He was under a lot of pressure from his board to resign.

Explanation:

1. The first sentence is hinting towards the arrival of something at a particular place. Hence,
‘at’ is the correct answer.

2. The second sentence is indicating a place at which an action took place. Hence, from is the
right answer.

3. The third sentence is expressing the situation in which a person is forced. Hence, under is
the right answer.

Therefore, the correct answer is option D.

25. D
Sol.
The correct sentences are:

1. Shane was riding on a horse’s back when the accident took place.

2. Riya was not allowed to let anyone in.

3. His floor was below mine.

Explanation:

1. Since Shane was riding a horse, he must have been in contact with the horse. Hence, ‘on’
is the right answer.

2. Since Riya was not allowed to come with people to a place, the correct answer, in this case,
is in.
www.byjusexamprep.com

3. Since he lived on the ground floor, he must be living at the lowest level. Hence, below is the
right answer.

Hence, option D is the right answer.

26. D
Sol.
The preposition from is used to showcase something that you want to avoid. Here, the CEO
wants to avoid others crippling her head. Therefore, the most appropriate preposition would be
from.

Hence, option D is the correct answer.

27. C
Sol.
Carouse: a noisy, lively drinking party

Overlook: to forgive or pretend not to notice something

Frolic: play or move about in a cheerful and lively way

Disrespect: pay no attention to

Demonic: relating to or characteristic of demons or evil spirits.

Therefore, frolic is similar in meaning to carouse.

Hence, option C is the correct answer.

28. A
Sol. Let's learn the meaning of the given words:

Benediction: the utterance of a blessing; a prayer asking for divine blessing


www.byjusexamprep.com

Criticism: the expression of disapproval of someone or something on the basis of perceived faults or
mistakes

Invocation: a prayer asking God for help, forgiveness, and more, especially as part of a religious service

Beatitude: supreme blessedness; complete happiness that comes from being blessed

Consent: approval or praise

Compliment: a polite expression of praise or admiration

Hence, option A is the correct answer.


29. B
Sol.
Baseness means a lack of moral principles.

Honour means to have high respect or regard for someone.

Disgrace means a state of shame.

Lowness means a position of inferior status.

Probity means the quality of having strong moral principles, honesty, and decency.

Inundation means an overwhelming abundance of people or things.

Hence, option B is the correct answer.

30. D
Sol.
Adjourn means to break off a meeting, legal case, or game with the intention of resuming it
later.

Prorogue means to discontinue a session of a parliament or other legislative assembly without


dissolving it.
www.byjusexamprep.com

Recess means a period of time when the proceedings of a parliament, committee, court of law,
or other official bodies are temporarily suspended.

Dangle means to temporarily prevent from continuing or being in force or effect.

Convene means to come together for a meeting or activity.

Contravene means to offend against the prohibition or order of (a law, treaty, or code of
conduct).

Hence, option D is the correct answer.

31. B
Sol.
The meaning of the difficult words given in the answer options are as follows:

Wither means to lose liveliness, force, or freshness.

Vanish means to cease to be visible.

Defraud means to rob by the use of trickery or threats.

Exhume means to remove from the place of burial or to unearth something.

Excessive inflation can lead to a burst. Similarly, excessive pulling can cause a tear. Option A
can be eliminated, as we cannot say that a thing losing its liveliness will cease to exist.
Options C, D, and E can be easily eliminated.

Hence, option B is the correct answer.

32. D
Sol.
The meaning of the difficult words given in the answer choices are as follows:

Brusque means abrupt.


www.byjusexamprep.com

Austere refers to self-denial.

Steadfastness means loyalty in the face of trouble and difficulty.

Droll means something that is comical in an odd or whimsical manner.

Comicalness means the quality or an act of being funny.

Nonchalant means having or showing a lack of interest or concern.

Temper: the degree of hardness and elasticity in steel or other metals.

Being rash means acting or doing without careful consideration of the possible consequences;
impetuous.

The meaning of flippant is not showing a serious or respectful attitude. So, flippant is the
opposite of respectful. Similarly, nonchalant is the opposite of concerned. Option A can be
eliminated, as brusque is not the opposite of ignorance. Similarly, option B can also be
eliminated. Option C shows a synonymous relationship. Rash provokes promptness and,
hence, is not similar to the relationship given. Thus, option D is the correct answer. Option E
shows the word on the left to be the property of the word on the right. Therefore, it is rejected.

Hence, the correct answer is option D.

33. B
Sol.
The massacre still horrifies people, but it will not horrify the people of the coming generations.
It has to be because the event is slowly being eradicated from their history. If the event is not
documented in their history books, the coming generations will not read it, and it will slowly
disappear. Therefore, documentation is the correct fit for the blank.

Hence, option B is the correct answer.

34. D
www.byjusexamprep.com

Sol. The meaning of the difficult words given in the answer options are as follows:

Fringe means to serve as a border for something.

Since there is gold in the semiconductor chips, and we dispose them off, we are disposing off
a lot or plenty of gold. The rest of the options can be eliminated. Now, the methods on which
they are working may not have been profitable.

Hence, option D is the correct answer.

35. A
Sol.
The meaning of the difficult words given in the answer options are as follows:

Reviled means to speak about someone or something in a very critical or insulting way.

The sentence has a contrasting marker which is although. So, the idea flow has to change.
Since his elegance is admired, he is not admired for his technical skills by many people.
Options B and C can be easily eliminated. Now, between options A and D, option D is a better
fit. If only a few people criticised him for his technical skills, then the sentence should not have
started with although.

Hence, option A is the correct answer.

36. D
Sol.
The correct answer is option D. The blank needs to be filled by the past form of the verb since
it is being preceded by the auxiliary ‘were’. The only verb in the past form given in the options
is tempted. Hence, option D the correct answer.

Tempt means to want something or to want to do something.

Hence, the correct answer is option D.


www.byjusexamprep.com

37. E
Sol. The data is insufficient since the number of matches to be played by India this year is not
given (we cannot assume that they will play 92 matches this year too).
Hence, option E is the correct answer.
38. C
Sol. Let’s assume there are ‘n’ MCQs on the test.
So, number of TITA questions = 20 – n
So, 5n + (20 – n)10 = 130

So, number of TITA questions = 20 – 14 = 6


Hence, option C is the correct answer.
39. D
Sol.

Let the number of girls be ‘n’.

So, amount of food consumed by all boys = 6 10 = 60 kg

Consumption by all girls = n 7 = 7n

So,

Hence, option D is the correct answer.


40. C
Sol. For one pair of yellow sandals, we require one left yellow and one right yellow sandal.
Consider the worst case scenario:
First 8 left yellow sandals; then 6 left blue sandals; then 6 right blue sandals; and then 1 right
yellow sandal: 8LY + 6LB + 6RB + 1RY or
First 8 right yellow sandals; then 6 left blue sandals; then 6 right blue sandals; and then 1 left
yellow sandal: 8RY + 6LB + 6RB + 1LY = 21 sandals
Hence, option C is the correct answer.
www.byjusexamprep.com

41. E
Sol. Let ‘s’ be the speed of the third jeep.

Speed of the second jeep = km/h

After half an hour, the first jeep has covered km

The second jeep has covered km


Now, according to the question:

s = 60 or 100/3
As ‘s’ must be greater than 50, we consider s = 60 km/h
Hence, option E is the correct answer.
42. B
Sol. Let the roots be x and x + 3.

Then, x + x + 3 = b

…(1)
x(x + 3) = c + 3x = c …(2)
Substitute the value of x from eq (1) in eq (2):

Hence, option B is the correct answer.


43. C
Sol.
Upstream speed of Boat E = = 16 km/h
www.byjusexamprep.com

Speed of the river = 20 – 16 = 4 km/h

Downstream speed of Boat B = 12 + 4 = 16 km/h

Time required for Boat B = = 5 hours

Hence, option C is correct.

44. E
Sol. Speed of Boat C downstream = 10 + 2 = 12 km/h

Speed of Boat D downstream = 18 + 2 = 20 km/h

Let the distance travelled by Boat C downstream be x km. The distance travelled by Boat D be
(300 – x) km.

Now,

2x + 900 = 1200

x = 150

Distance travelled by Boat D downstream = 300 – x = 300 – 150 = 150 km

Hence, option E is correct.

45. A
Sol. Boat F beats Boat A by either 70 km or 5 hours.

So, speed of Boat A upstream = = 14 km/h

Therefore, speed of stream = 15 – 14 = 1 km/h

Time taken by Boat A to travel 280 km upstream = = 20 hours


www.byjusexamprep.com

Distance covered by Boat C upstream in 20 hours = 20 × (10 – 1) = 180 km

Distance by which Boat A beats Boat C in the race = 280 – 180 = 100 km

Hence, option A is correct.

46. D
Sol. Downstream speed of Boat B = 12 + 4 = 16 km/h

Time taken by Boat B downstream for 112 km = =7h

Speed of Boat C downstream = 10 + 4 = 14 km/h

Time taken by Boat C downstream for 112 km = =8h

Therefore, Boat B takes less time than Boat C.

Required percentage = = 12.5%

Hence, option D is correct.

47. D
Sol. ( )=
On being divided by 7:
Rem =1+1=2
Hence, option D is the correct answer.
48. C
Sol. Total cost incurred = 6 lakhs
The company distributed 600 copies for free, so the number of copies remaining = 6000 – 600
= 5400

So,
www.byjusexamprep.com

Total revenue = 1200 180 + 4200 240 – Vendor’s discount of 50%


= 12.24 lakhs – 6.12 lakhs = 6.12 lakhs

So, profit% =
Hence, option C is the correct answer.
49. D
Sol. Let the incomes of A, B, and C be 3x, 5x, and 8x, respectively.
Let their spendings be 8y, 9y, and 10y, respectively.

So, A’s savings =


So, it implies x = 3x – 8y 8y = 2x x = 4y
So, the incomes of A, B, and C in terms of ‘y’ are 12y, 20y, and 32y, respectively.
Hence, their savings = (12y – 8y) : (20y – 9y) : (32y – 10y) = 4y : 11y : 22y
= 4 : 11 : 22
Hence, option D is the correct answer.
50. D
Sol. From the given information, we can have the figure given below:
www.byjusexamprep.com

Now, in order to maximise the number of people who did not eat any of the three desserts, we
would need to use the minimum number of people inside the three circles, making sure all the
conditions are met.
So, if we make the following configuration as shown below, then all the conditions are met.

In this case, the number of people who ate Halwa is 200 more than the number of people who
ate Rasmalai, and the number of people who ate Kheer also sums up to 1450.
Hence, the maximum number of people who did not eat any of the three desserts = 2500 –
1450 = 1050
Hence, option D is the correct answer.
51. C
Sol. Let the delivery rate of the pump discharging the tank be ‘x’ litres per minute.

So, the rate of delivery of the pump filling the same tank =

So,

x=
Hence, option C is the correct answer.
www.byjusexamprep.com

52. C
Sol. By hit and trial, the value of (x, y) satisfying 3x + 4y = 103 will be (33, 1).
As y < 0, the next value of y is –2 and x = 37.
Further pairs will be obtained by adding 4 to x or subtracting 3 to y.
The pattern follows an arithmetic progression with 37 as the first term and 4 as the common
difference. Let the last term be 80 and the number of terms be ‘n’.

will be approximately equal to 11.


Hence, option C is the correct answer.
53. A
Sol. The table can be filled in the following manner:

Number of Xalo and Twoplus users in Delhi = 2250 + 180 = 2430

Percentage of Viva and Oppi users in Delhi = 26% + 20% = 46%

So, 100% – 46% = 54% = 2430

100% = = 4500 (total users of the four mobile phones)

Number of Viva phone users = = 1170

Number of Oppi phone users = = 900

Likewise, when we can calculate the data for other cities, we get the following data:
www.byjusexamprep.com

At least 750 mobile phone users of each brand are there only in Kolkata.

Hence, option A is the correct answer.

54. C
Sol. The table can be filled in the following manner:

Number of Xalo and Twoplus users in Delhi = 2250 + 180 = 2430

Percentage of Viva and Oppi users in Delhi = 26% + 20% = 46%

So, 100% – 46% = 54% = 2430

100% = = 4500 (total users of the four mobile phones)

Number of Viva phone users = = 1170

Number of Oppi phone users = = 900

Likewise, when we can calculate the data for other cities, we get the following data:

From the above table, we find that Viva sold 9155 mobile phones in these five cities.

Hence, option C is the correct answer.

55. E
Sol. The table can be filled in the following manner:

Number of Xalo and Twoplus users in Delhi = 2250 + 180 = 2430


www.byjusexamprep.com

Percentage of Viva and Oppi users in Delhi = 26% + 20% = 46%

So, 100% – 46% = 54% = 2430

100% = = 4500 (total users of the four mobile phones)

Number of Viva phone users = = 1170

Number of Oppi phone users = = 900

Likewise, when we can calculate the data for other cities, we get the following data:

Required ratio = 4500 : 6000 : 2500 = 45 : 60 : 25 = 9 : 12 : 5

Hence, option E is the correct answer.

56. C
Sol. The table can be filled in the following manner:

Number of Xalo and Twoplus users in Delhi = 2250 + 180 = 2430

Percentage of Viva and Oppi users in Delhi = 26% + 20% = 46%

So, 100% – 46% = 54% = 2430

100% = = 4500 (total users of the four mobile phones)

Number of Viva phone users = = 1170


www.byjusexamprep.com

Number of Oppi phone users = = 900

Likewise, when we can calculate the data for other cities, we get the following data:

Required average = = 5400

Hence, option C is the correct answer.

57. E
Sol. Let the number of students in departments P, Q, and R be 4x, 5x, and 10x, respectively.

So, the number of females in department P =

The number of females in department Q =

The number of females in department R =


So, total number of females in all the departments = 3x + 3x + 8x = 14x

So, the required percentage =


Hence, option E is the correct answer.
58. B
Sol.
Number of ways of expressing 360 as a product of two factors =
Let N = 360 = × ×
Number of factors = (3 + 1)(2 + 1)(1 + 1) = 4 × 3 × 2 = 24

Number of ways in which 360 can be expressed as a product of two factors = = 12


www.byjusexamprep.com

Hence, option B is the correct answer.


59. E
Sol. As the person in Train B crosses Train A, the length of Train B is not considered.

Thus the total distance to be covered = Length of train A = 300 m

The word overtake implies that both the trains are moving in the same direction.

Relative speed = (72 – 36) km/h = 36 km/h = 36 × m/s = 10 m/s

So, time taken = seconds = 30 seconds

Hence, option E is the correct answer.


60. E
Sol.
Given: f(x2 – 9x + 12) = 5x + 9; we need to find the value of f(–6),
i.e., we want that value of x for which x2 – 9x + 12 = –6
Or x2 – 9x + 18 = 0
x2 – 6x – 3x + 18 = 0
x = 3, 6
Putting x = 3, 6 in the given function, we get
f(32 – 27 + 12) = 5(3) + 9 = 24
And f(36 – 54 + 12) = 5(6) + 9 = 39
Hence, option E is the correct answer.
61. C
Sol. Here, alligation has to be used twice as there are 3 components.
When A and C are mixed together to get Rs. 270:
www.byjusexamprep.com

So, the ratio of A : C = 5 : 6


And when B and C are mixed together to get Rs. 270:

The ratio of B : C =
Note: We cannot mix A and B here as 270 is not in between 210 and 230.
Now, these mixtures are added together in such a way that it satisfies one of the options.
Considering the 1st mixture, the ratio of A : B : C = 5 : 0 : 6
And taking thrice of the second mixture, A : B : C = 0 : 15 : 12
On adding both the ratios, we get 5 : 15 : 18
Hence, option C is the correct answer.
www.byjusexamprep.com

62. B
Sol. The table given below shows the marks obtained by all four students in three different
subjects:

Marks obtained by Arun in mathematics = 0.9 150 = 135

Similarly, the marks obtained by all the four students in each of the subjects can be calculated
and then the following table can be made.

Average marks obtained by all four students in mathematics = = 126

Average marks obtained by all four students in physics = = 91.5

Average marks obtained by all four students in chemistry = = 108.5

Required sum = 126 + 91.5 + 108.5 = 326

Hence, option B is correct.

63. C
Sol. The table given below shows the marks obtained by all four students in three different
subjects:

Marks obtained by Arun in mathematics = 0.9 150 = 135

Similarly, the marks obtained by all the four students in each of the subjects can be calculated
and then the following table can be made.
www.byjusexamprep.com

Required ratio = = = 15 : 11

Hence, option C is correct.

64. C
Sol. The table given below shows the marks obtained by all four students in three different
subjects:

Marks obtained by Arun in mathematics = 0.9 150 = 135

Similarly, the marks obtained by all the four students in each of the subjects can be calculated
and then the following table can be made.

Marks obtained by Varun in the practical exam of physics = 90 × = 27

Marks obtained by Sanjay in the practical exam of chemistry = 105 × = 21

Sum of the marks obtained in the practical exams = 27 + 21 = 48

Total marks obtained by Arun in English = 48 × = 60

Hence, option C is correct.


www.byjusexamprep.com

65. E
Sol. The table given below shows the marks obtained by all four students in three different
subjects:

Marks obtained by Arun in mathematics = 0.9 150 = 135

Similarly, the marks obtained by all the four students in each of the subjects can be calculated
and then the following table can be made.

Marks obtained by Vijay in mathematics = 135 × = 162

Marks obtained by Vijay in physics = 96 × = 72

Marks obtained by Vijay in chemistry = 119

Total marks obtained by Vijay in all three subjects = 162 + 72 + 119 = 353

Total marks in all three subjects = 150 + 120 + 140 = 410

Percentage of marks obtained by Vijay in all three subjects together = × 100 = 86% (after
neglecting the decimal part)

Percentage of marks obtained by Sanjay in all three subjects together

= × 100 = 84% (after neglecting the decimal part)

Required difference = 86 – 84 = 2%

Hence, option E is correct.


www.byjusexamprep.com

66. B
Sol. Each year has 52 weeks or 52 Saturdays.

Probability of a normal year having 53 Saturdays

Probability of a leap year having 53 Saturdays

The probability of a normal year and leap year is and , respectively.

The required probability

Hence, option B is the correct answer.


67. D
Sol. Let the cost price of the item be Rs. x.

Marked price of the item = Rs. 1.8x

According to the question,

x – (1.8x × 75% × 66.66%) = 320

⇒ x – 0.9x = 320

⇒ 0.1x = 320

⇒ x = 3200

Selling price = 3200 – 320 = Rs. 2880

Hence, option D is the correct answer.


68. B
Sol. Given: log (x2 – 11x + 25) = 0
7

x2 – 11x + 25 = 70
www.byjusexamprep.com

x2 – 11x + 24 = 0

(x – 3)(x – 8) = 0

x = 3 or x = 8

Now, log3 (27) = 3

And log8(72) = log89 + 1

Hence, option B is the correct answer.


69. A
Sol.
… (1)

… (2)
Subtracting (1) from (2), we get the following:

From (1),

P = Rs. 540
Then, 540 × r = 12000
R = 22.22%
Hence, option A is the correct answer.
70. C
Sol. Let’s assume pipes A and B can fill the tank without any leakage in ‘x’ minutes and ‘y’ minutes,
respectively.
According to the question,
x + y = 2 × 75 = 150 … (i)
And
www.byjusexamprep.com

+ =

⇒ =

⇒ =
⇒ xy = 3600 … (ii)
On solving equations (i) and (ii), we get the following:
Either x = 30 and y = 120 or x = 120 and y = 30
Pipe A takes 40 minutes to fill the tank when there is a leakage; so, x cannot be 120.
So, x = 30 and y = 120
Hence, option C is the right answer.
71. A
Sol. Using Statement I:
As N has 4 factors, the minimum number of factors = (1 + 1)(1 + 1)(1 + 1)(1 + 1)= 16
Using Statement II:
As we don't know the number of prime factors, it is not possible to derive the number of
factors.
Hence, option A is the correct answer.
72. E
Sol. Using Statement I: No information regarding the duration and combined profit; insufficient data
Using Statement II: No information regarding the investments and combined profit; insufficient
data
Using both the statements : No information regarding the number of investors; insufficient data
Hence, option E is the correct answer.
73. E
Sol. In each square of the diagram, the sum of numbers is always 57.

So, required number = 57 – (25 + 16 + 9) = 7

Hence, option E is correct.


www.byjusexamprep.com

74. B
Sol. In each group of three numbers, the lower number equals the average of the top two numbers.

Required number = =6

Hence, option B is correct.

75. C
Sol. Both the courses of action address the problem at hand. First of all, since Ankur is confused
about changing his field of work, he needs to do some research and figure out his field of
interest. Then, he needs to study about his field of interest to get the desired job. Hence,
option C is the right answer.
76. C
Sol. If the situation worsens due to the floods, the military must be deployed to take over rescue
operations from local governing bodies. The military have a wide range of expertise in
handling tough situations like floods. So, their expertise will help diffuse the situation in a much
better way. Also, II is also a valid course of action because people at low-lying areas are much
more susceptible to floods. Hence, they need to be evacuated immediately. Hence, option C is
correct.
77. B
Sol. I is more of an opinion than a course of action. Besides, we can’t say that the Reserve Bank
has no right to hike benchmark interest rates because the Reserve Bank handles these kinds
of things, and it is within their powers to do so. On the other hand, II is a valid course of action
that talks about what should be done in the midst of a hike in benchmark interest rates to
boost economic growth. Hence, option B is the right answer.
78. A
Sol. Since fuel prices are going off the charts, the governments must increase subsidies on fuel
prices to keep them under control and, thus, reduce inflation as much as possible. Hence, I is
www.byjusexamprep.com

a valid course of action. On the other hand, II is out of context of the given statement, and
hence it is rejected.
Hence, the correct answer is option A.
79. E
Sol.
Step I: All the letters are moved two places ahead according to their position in the English
alphabetical order.
Step II: The letters are arranged alphabetically within the words from left to right.
Step III: Each word’s letters are replaced with their opposite/reverse letters according to the
position of the English alphabetical order.
Step IV: Add the place values (the position in the English alphabetical order) of the two letters
in the middle.
Step V: Place the alphabet according to English alphabetical order corresponding to the value
of the sum of the digits of each of the numbers in the previous step.

Input: CDFV BGHN MJKL QWER TAUI


Step I: EFHX DIJP OLMN SYGT VCWK
Step II: EFHX DIJP LMNO GSTY CKVW
Step III: VUSC WRQK ONML THGB XPED
Step IV: 40 35 27 15 21
Step V: D H I F C
Hence, 15 is the smallest number in Step IV.
Hence, option E is the correct answer.

80. B
Sol.
Step I: All the letters are moved two places ahead according to their position in the English
alphabetical order.
Step II: The letters are arranged alphabetically within the words from left to right.
Step III: Each word’s letters are replaced with their opposite/reverse letters according to the
position of the English alphabetical order.
www.byjusexamprep.com

Step IV: Add the place values (the position in the English alphabetical order) of the two letters
in the middle.
Step V: Place the alphabet according to English alphabetical order corresponding to the value
of the sum of the digits of each of the numbers in the previous step.

Input: CDFV BGHN MJKL QWER TAUI


Step I: EFHX DIJP OLMN SYGT VCWK
Step II: EFHX DIJP LMNO GSTY CKVW
Step III: VUSC WRQK ONML THGB XPED
Step IV: 40 35 27 15 21
Step V: D H I F C
Hence, ‘GSTY’ is second from the right end in Step III.
Hence, option B is the correct answer.

81. B
Sol. Step I: All the letters are moved two places ahead according to their position in the English alphabetical
order.
Step II: The letters are arranged alphabetically within the words from left to right.
Step III: Each word’s letters are replaced with their opposite/reverse letters according to the position of the
English alphabetical order.
Step IV: Add the place values (the position in the English alphabetical order) of the two letters in the
middle.
Step V: Place the alphabet according to English alphabetical order corresponding to the value of the sum
of the digits of each of the numbers in the previous step.

Input: CDFV BGHN MJKL QWER TAUI


Step I: EFHX DIJP OLMN SYGT VCWK
Step II: EFHX DIJP LMNO GSTY CKVW
Step III: VUSC WRQK ONML THGB XPED
Step IV: 40 35 27 15 21
Step V: D H I F C
www.byjusexamprep.com

Hence, Step IV's first and last elements sum up to 61.


Hence, option B is the correct answer.
82. A
Sol. Step I: All the letters are moved two places ahead according to their position in the English alphabetical
order.
Step II: The letters are arranged alphabetically within the words from left to right.
Step III: Each word’s letters are replaced with their opposite/reverse letters according to the position of the
English alphabetical order.
Step IV: Add the place values (the position in the English alphabetical order) of the two letters in the
middle.
Step V: Place the alphabet according to English alphabetical order corresponding to the value of the sum
of the digits of each of the numbers in the previous step.

Input: CDFV BGHN MJKL QWER TAUI


Step I: EFHX DIJP OLMN SYGT VCWK
Step II: EFHX DIJP LMNO GSTY CKVW
Step III: VUSC WRQK ONML THGB XPED
Step IV: 40 35 27 15 21
Step V: D H I F C
Step II: EFHX DIJP LMNO GSTY CKVW
By arranging the words in alphabetical order from right to left, we get the following:
Step II: CKVW DIJP EFHX GSTY LMNO
Hence, option A is the correct answer.
83. B
Sol. In the first line itself, the author has mentioned that India’s greatest treasure is its human
resource, particularly women and children. So, to preserve this treasure, the government
needs to provide them with basic nutritional needs to aid their prosperity. Hence, argument II is
strong.
However, the same can’t be said about argument I because it is asking a question instead of
strengthening the case of helping out the underprivileged sections of the society. This makes
argument I sound like a burden rather than a responsibility.
Hence, option B is the right answer.
www.byjusexamprep.com

84. A
Sol. The intention behind the government's decision to make admissions free in museums is to
preserve and propagate our cultural heritage. Hence, I is a strong argument. Here, money is
not a factor because if it was the case, then the government would not have made entry into
the museums free.
Hence, option A is the right answer.
85. E
Sol. The logic for the given arrangement is as follows:
Input: 8743 9617 7381 3286 4511 7843
Step I: First two-digit number × Last two-digit number
For instance, 8743 → 87 × 43 → 3741
Step I: 3741 1632 5913 2752 495 3354
Step II: Digits of each number derived in Step I are written in reverse order (from right to left by keeping
the sequence same).
For instance, 3741 → 1473
Step II: 1473 2361 3195 2572 594 4533
Step III: Sum all the digits of the numbers derived in Step II.
For instance, 1473→ 1 + 4 + 7 + 3 → 15
Step III: 15 12 18 16 18 15
Step IV: Addition and subtraction of 7 are performed alternatively to the numbers by starting with addition
to the first number.
For instance, 15 + 7 = 22 and 12 − 7 = 5
Step IV: 22 5 25 9 25 8
Step V: Take a square of each number derived in Step IV and then add the same number to it to get the
corresponding numbers in Step V.
For instance, 22 → (22)2 + 22 →484 + 22 → 506
Step V: 506 30 650 90 650 72

Now, we get the following:

Input: 7453 2354 7492 6484 9673 4535


Step I: 3922 1242 6808 5376 7008 1575
Step II: 2293 2421 8086 6735 8007 5751
www.byjusexamprep.com

Step III: 16 9 22 21 15 18
Step IV: 23 2 29 14 22 11
Step V: 552 6 870 210 506 132

There are a total of six numbers in any step, which is an even number. So, two numbers are in the
middle.Therefore, two numbers in the middle of Step III are 22 and 21.
Hence, option E is the correct answer.
86. D
Sol. The logic for the given arrangement is as follows:
Input: 8743 9617 7381 3286 4511 7843
Step I: First two-digit number × Last two-digit number
For instance, 8743 → 87 × 43 → 3741
Step I: 3741 1632 5913 2752 495 3354
Step II: Digits of each number derived in Step I are written in reverse order (from right to left by keeping
the sequence same).
For instance, 3741 → 1473
Step II: 1473 2361 3195 2572 594 4533
Step III: Sum all the digits of the numbers derived in Step II.
For instance, 1473→ 1 + 4 + 7 + 3 → 15
Step III: 15 12 18 16 18 15
Step IV: Addition and subtraction of 7 are performed alternatively to the numbers by starting with addition
to the first number.
For instance, 15 + 7 = 22 and 12 − 7 = 5
Step IV: 22 5 25 9 25 8
Step V: Take a square of each number derived in Step IV and then add the same number to it to get the
corresponding numbers in Step V.
For instance, 22 → (22)2 + 22 →484 + 22 → 506
Step V: 506 30 650 90 650 72

Now, we get the following:

Input: 7453 2354 7492 6484 9673 4535


Step I: 3922 1242 6808 5376 7008 1575
Step II: 2293 2421 8086 6735 8007 5751
www.byjusexamprep.com

Step III: 16 9 22 21 15 18
Step IV: 23 2 29 14 22 11
Step V: 552 6 870 210 506 132
Therefore, 2293 2421 8086 6735 8007 5751 is Step II of the arrangement.
Hence, option D is the correct answer.
87. E
Sol. The logic for the given arrangement is as follows:
Input: 8743 9617 7381 3286 4511 7843
Step I: First two-digit number × Last two-digit number
For instance, 8743 → 87 × 43 → 3741
Step I: 3741 1632 5913 2752 495 3354
Step II: Digits of each number derived in Step I are written in reverse order (from right to left by keeping
the sequence same).
For instance, 3741 → 1473
Step II: 1473 2361 3195 2572 594 4533
Step III: Sum all the digits of the numbers derived in Step II.
For instance, 1473→ 1 + 4 + 7 + 3 → 15
Step III: 15 12 18 16 18 15
Step IV: Addition and subtraction of 7 are performed alternatively to the numbers by starting with addition
to the first number.
For instance, 15 + 7 = 22 and 12 − 7 = 5
Step IV: 22 5 25 9 25 8
Step V: Take a square of each number derived in Step IV and then add the same number to it to get the
corresponding numbers in Step V.
For instance, 22 → (22)2 + 22 →484 + 22 → 506
Step V: 506 30 650 90 650 72

Now, we get the following:

Input: 7453 2354 7492 6484 9673 4535


Step I: 3922 1242 6808 5376 7008 1575
Step II: 2293 2421 8086 6735 8007 5751
Step III: 16 9 22 21 15 18
www.byjusexamprep.com

Step IV: 23 2 29 14 22 11
Step V: 552 6 870 210 506 132

Sum = 23 + 2 + 29 + 14 + 22 + 11 = 101
Therefore, 101 is the sum of all the numbers in Step IV.
Hence, option E is the correct answer.
88. C
Sol. The logic for the given arrangement is as follows:
Input: 8743 9617 7381 3286 4511 7843
Step I: First two-digit number × Last two-digit number
For instance, 8743 → 87 × 43 → 3741
Step I: 3741 1632 5913 2752 495 3354
Step II: Digits of each number derived in Step I are written in reverse order (from right to left by keeping
the sequence same).
For instance, 3741 → 1473
Step II: 1473 2361 3195 2572 594 4533
Step III: Sum all the digits of the numbers derived in Step II.
For instance, 1473→ 1 + 4 + 7 + 3 → 15
Step III: 15 12 18 16 18 15
Step IV: Addition and subtraction of 7 are performed alternatively to the numbers by starting with addition
to the first number.
For instance, 15 + 7 = 22 and 12 − 7 = 5
Step IV: 22 5 25 9 25 8
Step V: Take a square of each number derived in Step IV and then add the same number to it to get the
corresponding numbers in Step V.
For instance, 22 → (22)2 + 22 →484 + 22 → 506
Step V: 506 30 650 90 650 72

Now, we get the following:

Input: 7453 2354 7492 6484 9673 4535


Step I: 3922 1242 6808 5376 7008 1575
Step II: 2293 2421 8086 6735 8007 5751
Step III: 16 9 22 21 15 18
www.byjusexamprep.com

Step IV: 23 2 29 14 22 11
Step V: 552 6 870 210 506 132
The smallest and the largest number in Step V are 6 and 870, respectively.
Hence, the required difference is (870 – 6) = 864
Hence, option C is the correct answer.
89. D
Sol. Conclusions:
I. No pay is a post: There is no relation between pay and post, so it is false.
II. Some cards are not pins: There is no relation between cards and pins, so it is false.
Hence, neither conclusion I nor II follows.
90. B
Sol. From the context, we can infer that differences of opinions between the state and the central
governments are quite common in the country. Hence, option B is the correct answer.
Option A is quite an extreme answer option because having a confrontation and fighting a war
are totally different things. Option C is incorrect because such incidents where centre and
states come face to face are quite common. Option D is irrelevant to the given context. Option
E takes a whole other route and talks nothing about the given context.
91. B
Sol. Since the ban on Russian football teams and representatives has come under the backdrop of
Russia’s invasion of Ukraine, it can be inferred that FIFA banning Russia’s football teams and
representatives is FIFA’s way of saying that Russia’s invasion of Ukraine must not be pursued
any further.
Option A gives an analogy that is extreme. This analogy cannot be inferred.
Option C is too extreme an opinion.
Option D cannot be inferred with any degree of certainty.
Hence, option B is the correct answer.
92. D
Sol. The author concludes that the National Power Corporation has reduced its emissions to the
maximum level possible by using one method. So, the assumption is that the company will not
be in a position to reduce emissions any further.
www.byjusexamprep.com

Hence, option D is the correct answer.


93. C
Sol. The author’s argument is that computer games hamper interpersonal skills. Children who
spend their free time playing computer games do not have interpersonal interaction with
people and have poorer communication skills, unlike other children.
So, the assumption is that other children who do not play computer games interact with people
in their free time.
Hence, option C is the correct answer.
94. A
Sol. The argument states that the defence expenditure can be cut further this year without any
effect on essential services. For this conclusion to be strengthened, we need an option that
will support the author’s claim that national security will not be affected by budget cuts.
Option A, if true, supports the claim as it states that the ministry has sufficient funds already.
Hence, the correct answer is option A.
95. C
Sol.
The one who likes kiwis sits second to the left of the G, who was born in August.

[There can be two possible cases here as G can face either north or south.]

J likes kiwis and is born in June.

Case 1:

Case 2:
www.byjusexamprep.com

Only three people sit between the one who likes kiwis and the one who likes mangoes.

[There can be two possible cases in each case.]

Case 1a:

Case 1b:

Case 2a:
www.byjusexamprep.com

Case 2b:

Immediate neighbours of the one who likes mangoes face the opposite directions.

Immediate neighbours of J face the opposite direction to that of J.

Case 1a:

Case 1b:

Case 2a:
www.byjusexamprep.com

Case 2b:

The one who likes litchis sits second from one of the ends of the line and is also born in
January.

J and the one who likes litchis are not immediate neighbours of each other.

Case 1a and Case 2a get eliminated here as there is no possible place left for the person who
likes litchi.

The one who likes papayas sits third to the left of J.

Case 1b:

Case 2b:
www.byjusexamprep.com

Immediate neighbours of J face the opposite direction to that of J.

In case 1b, J faces south hence immediate neighbours of J will face north.

In case 2b, J faces north hence immediate neighbours of J will face south.

Only three persons sit between the one who likes kiwis and K who sits second to the right of
the one who is born in May.

Case 1b:

Case 2b:

F, who likes bananas, is to the immediate right of H.


www.byjusexamprep.com

Case 1b:

Case 2b:

Immediate neighbours of the one who likes mangoes face the opposite direction with respect
to each other.

The one who was born in March sits exactly between G and K.

In both cases, the one who likes papayas is sitting exactly between G and K; hence, the one
who likes papayas is born in March.

The persons who are born in months following one another are not immediate neighbours of
each other.

From this statement, it can be concluded that immediate neighbours of both J and G are not
born in July; hence, the only person who was born in July is K.

Case 1b:
www.byjusexamprep.com

Case 2b:

Number of people sitting to the left of the one who likes pears is one less than the number of
persons sitting to the right of E.

(Only possible place for E and the one who likes Pear in both cases to satisfy the above
condition is that G likes Pear and E likes Papaya.)

Case 1b:

Case 2b:
www.byjusexamprep.com

E does not face south.

Case 1b gets eliminated here as E does not face south.

L likes grapes and does not sit at the extreme ends of the line.

The one who is born in September is not an immediate neighbour of G as G is born in August,
and it is mentioned in the question that persons who are born in months following one another
are not immediate neighbours of each other.

After filling the remaining information, we get the following:

Therefore, this is the final arrangement.


Therefore, F and the one who likes apples are sitting at the extreme ends of the line.

96. C
Sol.
The one who likes kiwis sits second to the left of the G, who was born in August.

[There can be two possible cases here as G can face either north or south.]

J likes kiwis and is born in June.

Case 1:
www.byjusexamprep.com

Case 2:

Only three people sit between the one who likes kiwis and the one who likes mangoes.

[There can be two possible cases in each case.]

Case 1a:

Case 1b:
www.byjusexamprep.com

Case 2a:

Case 2b:

Immediate neighbours of the one who likes mangoes face the opposite directions.

Immediate neighbours of J face the opposite direction to that of J.

Case 1a:

Case 1b:
www.byjusexamprep.com

Case 2a:

Case 2b:

The one who likes litchis sits second from one of the ends of the line and is also born in
January.

J and the one who likes litchis are not immediate neighbours of each other.

Case 1a and Case 2a get eliminated here as there is no possible place left for the person who
likes litchi.

The one who likes papayas sits third to the left of J.


www.byjusexamprep.com

Case 1b:

Case 2b:

Immediate neighbours of J face the opposite direction to that of J.

In case 1b, J faces south hence immediate neighbours of J will face north.

In case 2b, J faces north hence immediate neighbours of J will face south.

Only three persons sit between the one who likes kiwis and K who sits second to the right of
the one who is born in May.

Case 1b:

Case 2b:
www.byjusexamprep.com

F, who likes bananas, is to the immediate right of H.

Case 1b:

Case 2b:

Immediate neighbours of the one who likes mangoes face the opposite direction with respect
to each other.

The one who was born in March sits exactly between G and K.

In both cases, the one who likes papayas is sitting exactly between G and K; hence, the one
who likes papayas is born in March.
www.byjusexamprep.com

The persons who are born in months following one another are not immediate neighbours of
each other.

From this statement, it can be concluded that immediate neighbours of both J and G are not
born in July; hence, the only person who was born in July is K.

Case 1b:

Case 2b:

Number of people sitting to the left of the one who likes pears is one less than the number of
persons sitting to the right of E.

(Only possible place for E and the one who likes Pear in both cases to satisfy the above
condition is that G likes Pear and E likes Papaya.)

Case 1b:
www.byjusexamprep.com

Case 2b:

E does not face south.

Case 1b gets eliminated here as E does not face south.

L likes grapes and does not sit at the extreme ends of the line.

The one who is born in September is not an immediate neighbour of G as G is born in August,
and it is mentioned in the question that persons who are born in months following one another
are not immediate neighbours of each other.

After filling the remaining information, we get the following:

Therefore, this is the final arrangement.


‘K faces either north or south’ is the only true statement amongst the given statements with
respect to K.
www.byjusexamprep.com

97. A
Sol.
The one who likes kiwis sits second to the left of the G, who was born in August.

[There can be two possible cases here as G can face either north or south.]

J likes kiwis and is born in June.

Case 1:

Case 2:

Only three people sit between the one who likes kiwis and the one who likes mangoes.

[There can be two possible cases in each case.]

Case 1a:
www.byjusexamprep.com

Case 1b:

Case 2a:

Case 2b:

Immediate neighbours of the one who likes mangoes face the opposite directions.
www.byjusexamprep.com

Immediate neighbours of J face the opposite direction to that of J.

Case 1a:

Case 1b:

Case 2a:

Case 2b:
www.byjusexamprep.com

The one who likes litchis sits second from one of the ends of the line and is also born in
January.

J and the one who likes litchis are not immediate neighbours of each other.

Case 1a and Case 2a get eliminated here as there is no possible place left for the person who
likes litchi.

The one who likes papayas sits third to the left of J.

Case 1b:

Case 2b:

Immediate neighbours of J face the opposite direction to that of J.


www.byjusexamprep.com

In case 1b, J faces south hence immediate neighbours of J will face north.

In case 2b, J faces north hence immediate neighbours of J will face south.

Only three persons sit between the one who likes kiwis and K who sits second to the right of
the one who is born in May.

Case 1b:

Case 2b:

F, who likes bananas, is to the immediate right of H.

Case 1b:

Case 2b:
www.byjusexamprep.com

Immediate neighbours of the one who likes mangoes face the opposite direction with respect
to each other.

The one who was born in March sits exactly between G and K.

In both cases, the one who likes papayas is sitting exactly between G and K; hence, the one
who likes papayas is born in March.

The persons who are born in months following one another are not immediate neighbours of
each other.

From this statement, it can be concluded that immediate neighbours of both J and G are not
born in July; hence, the only person who was born in July is K.

Case 1b:

Case 2b:
www.byjusexamprep.com

Number of people sitting to the left of the one who likes pears is one less than the number of
persons sitting to the right of E.

(Only possible place for E and the one who likes Pear in both cases to satisfy the above
condition is that G likes Pear and E likes Papaya.)

Case 1b:

Case 2b:

E does not face south.

Case 1b gets eliminated here as E does not face south.

L likes grapes and does not sit at the extreme ends of the line.
www.byjusexamprep.com

The one who is born in September is not an immediate neighbour of G as G is born in August,
and it is mentioned in the question that persons who are born in months following one another
are not immediate neighbours of each other.

After filling the remaining information, we get the following:

Therefore, this is the final arrangement.


Therefore, ‘I, who likes apples, does not sit at the extreme ends of the line’ is the only false
statement amongst the given statements with respect to the given arrangement.

98. B
Sol.
The one who likes kiwis sits second to the left of the G, who was born in August.

[There can be two possible cases here as G can face either north or south.]

J likes kiwis and is born in June.

Case 1:

Case 2:
www.byjusexamprep.com

Only three people sit between the one who likes kiwis and the one who likes mangoes.

[There can be two possible cases in each case.]

Case 1a:

Case 1b:

Case 2a:
www.byjusexamprep.com

Case 2b:

Immediate neighbours of the one who likes mangoes face the opposite directions.

Immediate neighbours of J face the opposite direction to that of J.

Case 1a:

Case 1b:

Case 2a:
www.byjusexamprep.com

Case 2b:

The one who likes litchis sits second from one of the ends of the line and is also born in
January.

J and the one who likes litchis are not immediate neighbours of each other.

Case 1a and Case 2a get eliminated here as there is no possible place left for the person who
likes litchi.

The one who likes papayas sits third to the left of J.

Case 1b:

Case 2b:
www.byjusexamprep.com

Immediate neighbours of J face the opposite direction to that of J.

In case 1b, J faces south hence immediate neighbours of J will face north.

In case 2b, J faces north hence immediate neighbours of J will face south.

Only three persons sit between the one who likes kiwis and K who sits second to the right of
the one who is born in May.

Case 1b:

Case 2b:

F, who likes bananas, is to the immediate right of H.


www.byjusexamprep.com

Case 1b:

Case 2b:

Immediate neighbours of the one who likes mangoes face the opposite direction with respect
to each other.

The one who was born in March sits exactly between G and K.

In both cases, the one who likes papayas is sitting exactly between G and K; hence, the one
who likes papayas is born in March.

The persons who are born in months following one another are not immediate neighbours of
each other.

From this statement, it can be concluded that immediate neighbours of both J and G are not
born in July; hence, the only person who was born in July is K.

Case 1b:
www.byjusexamprep.com

Case 2b:

Number of people sitting to the left of the one who likes pears is one less than the number of
persons sitting to the right of E.

(Only possible place for E and the one who likes Pear in both cases to satisfy the above
condition is that G likes Pear and E likes Papaya.)

Case 1b:

Case 2b:
www.byjusexamprep.com

E does not face south.

Case 1b gets eliminated here as E does not face south.

L likes grapes and does not sit at the extreme ends of the line.

The one who is born in September is not an immediate neighbour of G as G is born in August,
and it is mentioned in the question that persons who are born in months following one another
are not immediate neighbours of each other.

After filling the remaining information, we get the following:

Therefore, this is the final arrangement.


Therefore, the one who likes pears sits third to the right of the one who is third to the left of the
one who likes pears.

99. D
Sol. The argument says that hospitals sell Tablet X for migraine and patients also take it. So, it is
an effective cure for migraine. We can weaken this argument by stating that the higher sale of
Tablet X is not due to its effectiveness but due to its accessibility—it is an over-the-counter
medicine. So, hospitals sell it more, and people take it more, but it does not necessarily mean
it is more effective.
Hence, option D is the correct answer.
100. C
Sol. A conclusion has to be drawn based on the facts given in the passage: the facts given are that
paracetamol prevents even a mild fever and mild fever is a symptom of many diseases. Also,
mild fever stimulates the production of disease-fighting white blood cells.
www.byjusexamprep.com

Taking both together, we can conclude that it prolongs a disease (the illnesses whose
symptoms are mild fever) by preventing mild fevers.
Hence, option C is the correct answer.
101. C
Sol. The software can detect forgeries without fail. Therefore, no one will be able to successfully
forge signatures in cheques and withdraw money from banks. Option C states this and is the
correct answer.
Options A, B, and D cannot be concluded from the passage. They are all outside the scope of
the passage.
Hence, the correct answer is option C.
102. B
Sol. Since it has been mentioned that unhealthy habits can reduce the efficacy of antibiotics, it can
be inferred that our health is dependent on how healthy we eat because the more unhealthy
food we eat, the more likely it is for us to be unaffected by antibiotics in case of bacterial
infections. This idea is best captured in option B, and hence it is the right answer.
103. A
Sol. Bhramar does not use the rackets of Brand Z.

Simar uses a racket of Brand Y. So, Simar does not use the rackets of Brand Z.

Shivar uses a racket of Brand X. So, Shivar does not use the rackets of Brand Z.

Hence, Amar uses a racket of Brand Z.

104. B
Sol. The pattern is as follows:

As,
www.byjusexamprep.com

And,

Similarly,

Hence, option B is correct.

105. D
Sol.
‘Prison’, ‘Pigeon’, and ‘Prism’ are three different elements. Hence, no relationship between these three
can be established.

106. D
Sol.
The venn diagram according to the given statements is as follows:
www.byjusexamprep.com

I. All tigers are kangaroos → It’s false as there is no relation between tigers and kangaroos.
II. ‘Only a few tigers are foxes’ is a possibility → ‘Some tigers being foxes and some tigers not being foxes’
is a possibility. Therefore, only conclusion II follows.
Hence, option D is the correct answer.
107. B
Sol. Since global crude oil prices have reduced around the world, the government can avail crude
oil at a knockdown price as opposed to the increased prices in which it was purchased before.
Hence, the Union Ministry can afford to cut fuel costs even in the wake of rising inflation
across the world. Hence, option B is the right answer here.
108. D
Sol. The Reserve Bank must have taken the decision to cut interest rates due to very valid
economic reasons. Therefore, this decision of the Reserve Bank is an effect of some cause.
Most small and medium enterprises are looking to hire people. This means they are
expanding, but we are not sure if it is due to the lower interest rates announced by the
Reserve Bank of India.
Therefore, both are independent effects.
Hence, the correct answer is option D.

You might also like